Главная Настройка Mobile Контакты NSFW Каталог Пожертвования Купить пасскод Pics Adult Pics API Архив Реквест доски Каталог стикеров Реклама
Доски


[Ответить в тред] Ответить в тред

Check this out!


[Назад][Обновить тред][Вниз][Каталог] [ Автообновление ] 525 | 70 | 101
Назад Вниз Каталог Обновить

Тред тупых вопросов №∞ (66) Аноним 28/02/17 Втр 19:55:44  328065  
1.jpg (46Кб, 700x525)
2.jpg (826Кб, 1920x1200)
3.jpg (22Кб, 480x399)
4.gif (367Кб, 600x600)
Тред вопросов о жизни, Вселенной и всем таком.

Спрашиваем то, за что в других местах выдают путёвку в биореактор. Здесь анонимные ученые мирового уровня критически рассмотрят любые гениальные идеи и нарисованные в Paint схемы.

Предыдущий тут: https://2ch.hk/spc/res/325429.html
Аноним 28/02/17 Втр 20:00:35  328069
93b8e1af5ebf79a[...].png (1166Кб, 1280x795)
Мне вот интересно, что будет на Земле, если Солнце вдруг погаснет полностью. Понятно, что пиздец, но как именно это будет происходить. Анон, опиши, если не лень, что со мной лично (Россия, Мухосрань) будет происходить через 10 минут, через 10 дней, через 10 месяцев и через 10 лет.
Аноним 28/02/17 Втр 20:08:39  328074
Википедия говорит, что температура экзосферы Земли составляет от 1500 до 3000 К, и что она не снижается с высотой, пока экзосфера не переходит в вакуум. Но ведь вакуум это для человека холодно, разве нет? Что будет чувствовать гипотетический всё чувствующий, но неубиваемый человек, если его будут постепенно поднимать от нижних слоев экзосферы до космического вакуума? Сначала адское жарище, потом постепенно холод?
Аноним 28/02/17 Втр 20:11:56  328075
>>328074
>Но ведь вакуум это для человека холодно
Вакуум для человека никак. Вакуум температуры не имеет.
Аноним 28/02/17 Втр 20:19:02  328076
>>328074
1500 К в очень разряженной среде далеко не равносильны 1500 К в условиях 1 атмосферы. Это просто показатель кинетической энергии частиц разряженной среды.
Аноним 28/02/17 Втр 20:22:37  328077
>>328069
в первые 8 минут ваще ничего не будет, так что будет время на то, что бы съебаца
Аноним 28/02/17 Втр 20:22:42  328078
>>328074
>температура экзосферы Земли составляет от 1500 до 3000 К
Эффекта от этого не почувствуешь - слишком редкие столкновения с частицами.
>Что будет чувствовать гипотетический всё чувствующий, но неубиваемый человек, если его будут постепенно поднимать от нижних слоев экзосферы до космического вакуума?
Его хватит тепловой удар от того, что тепло тела некуда будет скидывать, кроме как излучением в космос. Если это будет на солнышке, а не в тени - это произойдет ещё быстрее, представь себе полдень на экваторе.
Аноним 28/02/17 Втр 20:23:07  328079
Завтра иду на свидание с одноклассницей. Она псевдоинтеллектуалка (смотрит Ларина, читает Докинза и т.д.) Хочу впечатлить ее своими знаниями о космосе. Что ей рассказать?
Аноним 28/02/17 Втр 20:24:45  328080
>>328079
Пизды дам за Докинза.

Я как-то тянке загнал о том, что там, где есть звезды, может их уже и не быть, а если и есть, то могут распологаться и не там, где мы их видим. Зашло норм
Аноним 28/02/17 Втр 20:25:15  328081
>>328069
Есть вроде описание на ТВТропах, можешь поискать. Ну а так, вкратце

>через 10 минут
Ты охуеваешь, и все остальные тоже. На улице темно, на небе светят только звёзды.

>через 10 дней
Стало пиздец холодно, но люди умеют обогреваться. Фотосинтезирующие растения погибли (в твоей мухосрани на данный момент это комнатные и хвоя, остальные могли бы продержаться ещё несколько месяцев). Цены на отопление, электричество и еду выросли в разы. Разные учёные придумывают, как жить, чем греться, как синтезировать в больших количествах кислород.

>через 10 месяцев
Конкретно ты, скорее всего, мёртв. Как и подавляющее большинство людей. Оставшиеся живут примерно как жили бы люди на космической базе на Марсе, ну или скорее на Плутоне. Греются о ядерные реакторы. Строят упиздовалку к другой звезде.

>через 10 лет
При нашем уровне развития технологий, я думаю, на Земле мы бы столько не протянули. Хотя кто знает. Если было достаточно времени подготовить корабль, то уцелевшие улетают в другую систему.
Аноним 28/02/17 Втр 20:27:21  328082
>>328069
>происходить через 10 минут
Через 10 минут, точнее через 8 ты узнаешь, что Солнце погасло. Ничего необычного не произойдет. Просто внепланово настанет ночь. Безлунная, конечно.
>через 10 дней
Через 10 дней температура по всей планете станет одинаковой и будет равна 0 по Цельсию. Жратва в магазинах и на складах закончится. Люди начнут голодать и убивать друг-друга за еду. Начнутся голодные бунты и первые случаи каннибализма.
>через 10 месяцев
Закончатся последние запасы еды в крупных хранилищах, тем счастливчикам, что смогли в них укрыться предстоит медленно умирать. Ты в своем Мухосранске будешь к тому времени давно мертв.
>через 10 лет.
На поверхности планеты будет -120 градусов, все моря и океаны промерзнут на десятки метров. Единственная еще теплящаяся жизнь будет обитать в глубине океанов, возле геотермальных источников. При этом крупные формы исчезнут и там. Поверхность планеты абсолютно безжизненна.
Аноним 28/02/17 Втр 20:27:43  328083
>>328079
Расскажи легенду почему Млечный путь называли раньше Чумацкий шлях, всегда с бабами заходит.
Аноним 28/02/17 Втр 20:28:58  328085
>>328081
>как синтезировать в больших количествах кислород.

Нет нужды вообще в этом. Его хватит не на одну тысячу лет.
Аноним 28/02/17 Втр 20:29:18  328086
>>328081
>на Земле мы бы столько не протянули
Хотя не, хуйню написал. Уж если предполагать, что сможем корабль построить, то и здесь сможем торчать не меньше, чем он бы летел, то бишь лет сто минимум.
Аноним 28/02/17 Втр 20:30:09  328087
>>328080
Ну, это банальщина. Года 3 назад может быть еще прокатывало.
Докинз представляет интерес, только в качестве биолога. Слепой часовщик - его единственная нормальная книга.Сейчас он занят ничем, кроме пересказа баянов со времен средневековой схоластики. Серьезно, не понимаю людей, которые готовы так сильно поехать на теме спора с верующими (ладно Докинз, он хотя бы деньги на этом делает, но во его аудитория...)
Аноним 28/02/17 Втр 20:30:30  328088
Вопрос реально ОЧЕНЬ тупой, так что задам его здесь. На днях обмазался статьями про эволюцию Вселенной от БВ до наших дней, потом читал про эволюцию жизни на Земле, про геохронологическую шкалу от архея до голоцена... Читал всё это не первый раз, но на этот раз возникло какое-то ощущение искусственности, что ли, всего этого. Ну ведь реально, херня какая-то - бомбануло хуй пойми что и после этого оно начало развиваться по законам, строго заданным, видимо, ещё до начала всей этой хуйни, в итоге слеплись из всей этой мутной каши шарики-хуерики, которые слеплись в другие шарики или спиралики, только побольше. И всё было просто и понятно, но вот на одном из этих шариков после того, как в него врезался другой шарик, всё внезапно пошло вообще по ебанатскому сценарию: случайно образовались какие-то изолированные от среды самоклонирующиеся скопления вещества, со временем начавшие организовывать всё более сложные конструкции, и они не разрушились несмотря на всю череду лютых пиздецов, которые 100% должны были их уничтожить, и дошли до того, что сейчас я, их потомок, такой сижу и пишу обо всём этом и чувствую себя пупом земли. Да ну нахуй, как такое вообще случилось? Ебанулись, что ли?
Аноним 28/02/17 Втр 20:30:31  328089
>>328083
>Чумацкий шлях
Это только на Украине.
Аноним 28/02/17 Втр 20:30:48  328090
>>328089
Да похуй,везде заходит
Аноним 28/02/17 Втр 20:31:07  328091
>>328079
Будь романтиком, расскажи как там срут в пылесос и пьют воду, регенерированную из мочи.
Ещё можешь пояснить ей за требования к стыковочным узлам по International Docking System Standard, или обрисовать в общих чертах спецификации кабельной сети Протона, впечатлит наверняка.
Аноним 28/02/17 Втр 20:31:50  328092
>>328091
Годно.
Аноним 28/02/17 Втр 20:32:39  328093
>>328079

>Одно из наиболее впечатляющих открытий астрономии — это открытие источника энергии звезд, поддерживающего их горение. Один из тех, кто открыл это, отправился на прогулку с девушкой как раз в ночь после того, как понял, что на звездах происходит ядерная реакция, что в этом причина их свечения. Она сказала: «Взгляни, как чудесно сияют звезды!» А он ответил: «Да. Чудесно. А ведь сегодня я — единственный человек в мире, который знает, почему они сияют!» Она только рассмеялась. На нее не произвело впечатления, что он — единственный человек, понимающий, почему звезды светят. Что ж, как это ни печально, быть одиноким и непонятым — это в порядке вещей.

Ричард Фейнман
Фейнмановские лекции по физике, Том 1
Аноним 28/02/17 Втр 20:34:18  328094
>>328087
попробуй загнать про кротовые норы, альбукерки
Аноним 28/02/17 Втр 20:35:02  328095
>>328088
> как такое вообще случилось?
прост
Аноним 28/02/17 Втр 20:36:35  328096
>>328065 (OP)
нельзя было подождать пока прошлый укатится с нулевой?
Аноним 28/02/17 Втр 20:38:47  328097
>>328088
>как такое вообще случилось?
Потому что могло.
Аноним 28/02/17 Втр 20:39:49  328098
>>328096
Нельзя. Тебе не похуй? Подобные правила переката касались только огурцов из-за ебанистических скоростей ксп-тредов.
Аноним 28/02/17 Втр 20:39:55  328099
>>328096
Нет нахуя, что бы пришло несколько оболтусов из бэ, которые и нулевую до середины проскроллить не в состоянии и понасоздавали тредов в стиле "Анон, как срут в космосе?"
Аноним 28/02/17 Втр 20:49:31  328104
>>328085
Ну ок, да. И да, я не учёл теплового равновесия на время замерзания океанов, действительно какое-то время будет температура держатся у нуля. И ещё вулканы, будут люди жить у вулканов, гейзеров и всего такого какое-то время. И ещё будут поджигать выходы природного газа.
Аноним 28/02/17 Втр 20:55:56  328108
>>328065 (OP)
Можно ли быстрее света путешествовать?
Аноним 28/02/17 Втр 20:58:13  328109
>>328108
Можно упасть в пузырь альбукерке, наса уже почти надула его.
Аноним 28/02/17 Втр 20:58:51  328110
>>328108
Гугли пузырь алкубъерре.
Аноним 28/02/17 Втр 21:00:02  328112
1368314840.jpg (32Кб, 410x280)
Почему ученые продолжают отрицать дальнодействие после экспериментов с запутанными квантами? Можно сколько угодно кукарекать о том, что информация не передается и т. д., но ведь дальнодействие налицо, и оно явно ссыт на лицо утверждению, что скорость света есть максимальная скорость взаимодействия в физических процессах.
Аноним 28/02/17 Втр 21:01:49  328113
>>328112
>кукарекать
>оно явно ссыт на лицо
Выйди нахуй отсюда, животное, и зайди снова как человек.
Аноним 28/02/17 Втр 21:02:09  328114
>>328109
Ебанный стыд...
Во-первых, Алькубьерре.
Во-вторых, не упасть, а создавать вокруг корабля изнутри (иначе кина не будет).
В-третьих, НАСА искривляет пространство на десятимиллионную часть, контролируя это сверхточными интерферометрами, до самого варп-привода здесь - как до Антарктиды раком.
Аноним 28/02/17 Втр 21:02:57  328115
>>328112
Сколько уже информации передали быстрее скорости света?
Аноним 28/02/17 Втр 21:04:12  328116
>>328113
Ну хорошо, прекрасный сэр. Почему ученые продолжают отрицать дальнодействие после экспериментов с запутанными квантами? Можно сколько угодно оправдываться тем, что информация не передается и т. д., но ведь дальнодействие налицо, и оно явно противоречит утверждению, что скорость света есть максимальная скорость взаимодействия в физических процессах. Так лучше?
Аноним 28/02/17 Втр 21:06:47  328117
>>328079
Расскажи о чёрных дырах, о том, что они излучают тёмную энергию, которая захватывает свет своей гравитацией, о том, что во вселенной более трех тысяч галактик, и в каждой по паре миллионов звёзд, а в центрах сверхмассивные чёрные дыры, которые постепенно затягивают все звёзды в галактике. Расскажи о квазарах — огромных планетах, которые так сильно сжимаются под своим весом, что начинают светиться как звёзды, и отсюда их название — "подобные звёздам". Расскажи, как во время миссии Аполлона-13 им пришлось надевать скафандры и делать дырку в корабле, чтобы двигаться за счет воздуха, вылетающего оттуда.
Аноним 28/02/17 Втр 21:07:26  328118
>>328115
Да тут суть даже не в возможности передачи информации, а в том, что есть физически измеримая величина, которая передаётся мгновенно. Это ли не пиздец.
Аноним 28/02/17 Втр 21:08:38  328119
>>328118
Луч лазера по луне тоже может скользить быстрее света.
Точка пересечения огромных ножниц тоже может перемещаться быстрее света.
Тень тоже может перемещаться быстрее света.
И что? Что это даёт-то?
Информацию передать можешь?
Аноним 28/02/17 Втр 21:11:56  328122
601.jpg (10Кб, 620x250)
>>328109
>>328114
Я словно уже где-то видел этот диалог, и не раз.
Аноним 28/02/17 Втр 21:12:09  328123
>>328119
>Луч лазера по луне тоже может скользить быстрее света.
>Тень тоже может перемещаться быстрее света.
Шта? Тут поподробнее. Хуй с ней, с абстрактной точкой пересечения ножниц (это абстракция), но не верю, что реальная точка соприкосновения луча лазера с Луной или точка возлегания тени на поверхности может перемещаться быстрее света.
Аноним 28/02/17 Втр 21:14:23  328125
fukken lold2.png (71Кб, 600x400)
>>328123
>не верю
А что ей мешает, кроме твоего неверия?
Аноним 28/02/17 Втр 21:16:11  328126
>>328125
Ну в данном случае правильное рассмотрение задачи, где каждый фотон движется не быстрее скорости света.
Аноним 28/02/17 Втр 21:20:10  328128
>>328125
Блядь, с кем я общаюсь тут на спейсаче. Объясню, ладно.
Окей, изменил ты угол наклона лазера. Но твой лазерный пучок всё равно имеет скорость не больше скорости света, а то, что ты заранее рассчитал, куда он упадет, это лично твои представления.
То же самое с тенью. Переместил ты предмет, молодец. Но тень возляжет на поверхность только со скоростью света. В треде уже говорили, что даже в случае полного исчезновения Солнца Земля окутается во мрак только через 8 минут.

С запутанными квантами совсем ДРУГОЕ дело. Эти уроды меняют своё состояние действительно независимо от расстояния, иначе бы вокруг них не было столько ора и плясок.

Я хуею. Пришёл с тупым вопросом в тупой тред, а тут даже тупее меня. Пиздец.
Аноним 28/02/17 Втр 21:20:22  328129
Посоны, вот нам говорят что нейтронные звезды летят охуенно быстро, связано, дескать, с тем, что взрыв сверхновой неоднороден и ядро получает нехилый такой пинок. Иногда скорости настолько много, что ее хватает для того, чтобы встать на траекторию съеба из сраной галактики. А что насчет газа? Он ведь тоже получает огромное ускорение, но вместо того, чтобы улететь из галактики он остывает и кучкуется в туманностях. Почему так происходит?
Аноним 28/02/17 Втр 21:22:32  328130
>>328126
Ну так они и движутся не быстрее скорости света.
Другой пример.
Посмотри на Венеру. А потом переведи взгляд на Юпитер.
Поздравляю, тебе в глаз одновременно залетели фотоны испущенные с 10ае друг от друга!
И кто мешает в одночасье прийти в очень удаленные друг от друга точки фотонам испущенным из одного места?

>>328128
Я ПОНЯЛ, что кванты меняют состояния быстрее света.
Ты информацию-то ими передавать собираешься, или как? Потому, что если нет - то это ёбаный солнечный зайчик на луне.
Очевидная, блядь, аналогия, а этот не понял да ещё и обзываться стал.
Аноним 28/02/17 Втр 21:24:10  328131
>>328129
Обычно останки сверхновой остаются примерно там же торчать. Это когда бинарная система ёбает тогда импульс мощный.
А газики - они же легкие. Их даже давление света соседних звёзд может затормозить.
Вон у нас Солнце тысячами тонн атмосферу сдувает к хуям, например.
Аноним 28/02/17 Втр 21:24:10  328132
14882960211270.jpg (89Кб, 640x640)
>>328116

Не знаю, что ты имеешь ввиду конкретно под
>ученые продолжают отрицать
Какие ученые, когда и где они что-либо отрицали? Имена, фамилии, явки, нотариально заверенные скриншоты, предъявите, пожалуйста. А то вроде зашли в этот раз во фраке, а от вас ЗОГ-помойкой несет.
>Можно сколько угодно оправдываться тем, что информация не передается
Наука не стоит на месте, есть определенные надежды, что с помощью слабых измерений можно обойти теорему о запрете клонирования. Однако, стоит относится к таим вещам осторожно и внимательно погуглить схему квантовой телепортации, что бы понимать, почему и как.
А что касается
>явно противоречит утверждению
то квантовая нелокальность происходит исключительно в копенгагенской интерпритации КМ, что бы жопа не горела, можно рассматривать весь этот пиздец с точки зрения многомировой интерпритации и не ебать мозг ни себе, ни другим.
Аноним 28/02/17 Втр 21:25:29  328133
>>328130
>Я ПОНЯЛ, что кванты меняют состояния быстрее света.
Я просто хочу понять, почему это не пиздец принципу локальности, и в тред пришёл не ради того, чтобы кого-то оскорблять.
Аноним 28/02/17 Втр 21:28:11  328134
>>328131
> Обычно останки сверхновой остаются примерно там же торчать.
Да не, мы видим нейтронные звезды в остатках сверхновой потому, что они еще не успели улететь. То есть, сама сверхновая бахнула сотни и тысячи лет назад, а через миллион лет ядро сверхновой улетит, а оставшийся газ остынет и станет для нас невидимым.
Аноним 28/02/17 Втр 21:30:46  328136
>>328132
>квантовая нелокальность происходит исключительно в копенгагенской интерпритации КМ, что бы жопа не горела, можно рассматривать весь этот пиздец с точки зрения многомировой интерпритации и не ебать мозг ни себе, ни другим.
Что-то это вариативное восприятие реальности в лучших традициях гуманитарных наук напоминает подход "Я художник, я так вижу". И хуй поспоришь, ведь у каждого индивида и каждой школы есть собственный вариант толкования реальности. Что для одного зло, для другого добро. Не кажется, что для физики это немного унизительно, что ли?
Аноним 28/02/17 Втр 21:37:58  328138
Как бы выглядело небо, если бы Млечный Путь был шаровой галактикой, а Земля была бы в том же положении относительно центра, что и сейчас?
Аноним 28/02/17 Втр 21:38:23  328139

>>328136
Ты сейчас ведешь себя как типичный гумманитарий, пытаешься проецировать свой недостаток знаний в предмете, в которым ты шаришь не больше, чем свинья в апельсинах, на свое уютненькое виденье макромира. При этом ты даже не в курсе, что существуют различные интерпретации КМ, которые суть одной хуйни, просто в одном случае при измерении наступает коллапс квантового состояния, а в другом нет. При этом какой пользоваться - это дело вкуса и личного предпочтения.
Аноним 28/02/17 Втр 21:39:56  328140
>>328116
Никакого дальнодействия нет, так же, как нет его, если ты возьмёшь две бумажки, на одной напишешь "хуй", а на другой "пизда", отправишь не смотря случайную по почте своей мамке, потом откроешь свою и узнаешь, что пришло мамке.
Аноним 28/02/17 Втр 21:40:09  328141
>>328117
Аплодирую этому фантазеру стоя
Аноним 28/02/17 Втр 21:42:25  328142
>>328138
>Как бы выглядело небо, если бы Млечный Путь был шаровой галактикой, а Земля была бы в том же положении относительно центра, что и сейчас?

Space Engine всё еще можно скачать бесплатно, без СМС.
Аноним 28/02/17 Втр 21:43:05  328144
>>328138
Вместо Млечного Пути была бы Млечная Клякса.
Аноним 28/02/17 Втр 21:43:32  328145
>>328142
>Space Engine всё еще можно скачать бесплатно, без СМС.
Я под Линуксом. : (
Аноним 28/02/17 Втр 21:44:24  328146
>>328144
Кстати, слово Галактика имеет корень Лактос - молоко.
Интересно, как бы звучало это слово, если бы млечный путь назывался "сперма отца"?
Аноним 28/02/17 Втр 21:46:01  328149
>>328133
1.пока на этой квантовой поеботе не построят ПЕРЕМОГУ с нулевым пингом
2. когда построят - поменяют формулировку "информации" шоб никого не коробило, поскольку там речь шла о движении, чем эта ваша квантовая телепортация не особо является
3. всё
Аноним 28/02/17 Втр 21:46:57  328150
>>328145
>Я под Линуксом. : (
Сразу видно, что выёбывающийся хипстор и линь накатил чтобы быть нитакой какфсе.
Тру-линуксер уже через 5 секунд знает, что у СЕ рейтинг платиновый в вайне.
Аноним 28/02/17 Втр 21:49:00  328151
Если бы Земля была на орбите Сатурна, то нам было бы очень холодно?
Аноним 28/02/17 Втр 21:49:53  328152
>>328151
Представь себе.
Аноним 28/02/17 Втр 21:50:09  328153
>>328149
>1.пока на этой квантовой поеботе не построят ПЕРЕМОГУ с нулевым пингом
Когда построите - приходите.
>2. когда построят - поменяют формулировку "информации" шоб никого не коробило
Хуйня. Фундаментальная возможность передачи данных быстрее света нахуй перекосоёбит всю физику и даст миллиарды срачей, грантов, рисёчей и профитов.
>поскольку там речь шла о движении,
Не пизди чего не знаешь. О передаче информации выше скорости света. Хочешь - телепортацией, хочешь - пузырь альбукерке надувай.
Нарушишь принцип причинности, передавай привет своей бабу
Аноним 28/02/17 Втр 21:50:20  328154
>>328146

Pater sperma
Аноним 28/02/17 Втр 21:50:38  328155
>>328146
Как-то вроде Spermatoupatera
Аноним 28/02/17 Втр 21:50:45  328156
В одном фильме видел, что пыль и осколки, из которых и состоят кольца, движутся в разных направлениях? Это так или просто так мне показалось поскольку у более мелких частиц скорость быстрее.
Аноним 28/02/17 Втр 21:51:03  328157
>>328151
да, но и атмосфера была бы немного гуще, так что даже хуй знает насколько холоднее
Аноним 28/02/17 Втр 21:51:06  328158
>>328156
Кольца сатурна.
Аноним 28/02/17 Втр 21:51:55  328159
>>328153
> Нарушишь принцип причинности
покажи как он нарушается, выебос
Аноним 28/02/17 Втр 21:52:09  328160
>>328156
В одном направлении, с небольшой разницей в скоростях.
>Это так или просто так мне показалось
Показалось.
Аноним 28/02/17 Втр 21:54:07  328161
Если бы у Юпитера была ретроградная орбита, пошла бы по пизде вся Солнечная система или нет? Что бы поменялось?
Аноним 28/02/17 Втр 21:55:09  328163
>>328157
>да, но и атмосфера была бы немного гуще
Почему?
Аноним 28/02/17 Втр 21:55:35  328164
>>328161
>ретроградная орбита
Чего бля?
Аноним 28/02/17 Втр 21:55:35  328165
>>328159
Превышением скорости света.
Аноним 28/02/17 Втр 21:55:38  328166
>>328123
>не верю, что реальная точка соприкосновения луча лазера с Луной или точка возлегания тени на поверхности может перемещаться быстрее света
Светим лазерной указкой в звезду А. За пару секунд перенаводим указку на звезду Б, которая от нас на такой же дистанции, но за десятки парсек от А. Вуаля, зайчик преодолел десятки парсек за 2 секунды. С тенью аналогично.
https://ru.wikipedia.org/wiki/%D0%A4%D0%B0%D0%B7%D0%BE%D0%B2%D0%B0%D1%8F_%D1%81%D0%BA%D0%BE%D1%80%D0%BE%D1%81%D1%82%D1%8C
Аноним 28/02/17 Втр 21:56:15  328167
>>328163
меньше выносится излучением и прочими вспышками Солнца
Аноним 28/02/17 Втр 21:56:31  328168
>>328165
хуйня
Аноним 28/02/17 Втр 21:56:41  328169
>>328151
На орбите Сатурна например Титан. Можешь погуглить, какой пиздатый там курорт.
Аноним 28/02/17 Втр 21:57:31  328170
>>328164
Ты не знаешь, что такое ретроградная орбита?
Аноним 28/02/17 Втр 21:57:43  328171
>>328169
Титан нещитово, это противоположность Венеры - у него очень сильный антипарниковый эффект. Так-то там потеплее было бы.
Аноним 28/02/17 Втр 21:57:49  328172
>>328166
Ты нихуя не понял.
Аноним 28/02/17 Втр 21:57:50  328173
>>328169
Я на титане был в IW) Охуительные пейзажи, можешь глянуть на ютабе)
Аноним 28/02/17 Втр 21:58:03  328175
>>328161
Особенно ничего, хотя там может хуйня какая случилась с орбитальными резонансами.
Но откуда бы Юпитеру взять ретроградное обращение?
Аноним 28/02/17 Втр 21:59:15  328177
>>328175
>Но откуда бы Юпитеру взять ретроградное обращение?
Ну, почему бы и нет.
Аноним 28/02/17 Втр 21:59:29  328178
>>328161
Если бы она вдруг поменялась - устойчивость СС бы нарушилась. Если она такой была бы изначально - ну просто на ранней стадии существования СС планеты бы отфильтровались чуть иначе, но то или иное равновесие было бы достигнуто, точно так же как и сейчас.

>>328172
Чего именно? Я отвечал на процитированное.
Аноним 28/02/17 Втр 22:00:31  328179
>>328171
>anti-greenhouse effect due to the haze reduces the surface temperature by 9 K
Тююю, несчитова ему. Ну окей, как на Титане, но на 10 градусов теплее.
Аноним 28/02/17 Втр 22:02:25  328180
>>328177
Потому что этот толстый пидорас образовался из протопланетарного облака, у которого было конкретное направление вращения. Так что как раз твердое НЕТ.
Аноним 28/02/17 Втр 22:03:25  328182
>>328179
ещё добавь положительный парниковый земляшки и её повышенную из-за удаления от солнца атмосферу
Аноним 28/02/17 Втр 22:03:25  328183
А если бы Земляшка была как Уран, катилась на боку, что-то поменялось бы существенно?
Аноним 28/02/17 Втр 22:04:21  328185
>>328161
Устойчивость солнечной системы обусловлена тем, что всё неустойчивое говно попадало друг на друга давным-давно, остались только те, кому повезло. Выводы делай сам.
Аноним 28/02/17 Втр 22:05:15  328186
>>328183
сезоны и климат
Аноним 28/02/17 Втр 22:05:45  328187
>>328183
Для земляшки да, было бы лето на одном полюсе и зима на другом. Остальным было бы похуй.
Аноним 28/02/17 Втр 22:07:02  328188
>>328182
На земляшке очень разная температура. От +56 до -89, так что хватит торговаться.
Аноним 28/02/17 Втр 22:07:58  328189
>>328187
Почему тогда Уран так гладко выглядит? При постоянно обработке Солцнем с одного края и холоде в другом я ожидал увидеть какие-то визуальные различия. А тут тупо зеленый однородный шар.
Аноним 28/02/17 Втр 22:08:05  328190
>>328188
Это гораздо меньшая разница, чем у любого другого бессезонного/безатмосферного тела.
Аноним 28/02/17 Втр 22:08:20  328191
>>328187
там не приливной захват
там всё сложно
Аноним 28/02/17 Втр 22:08:57  328192
>>328188
чё, на Титане везде одинаковая?
Аноним 28/02/17 Втр 22:10:43  328194
>>328189
Потому это газовый гигант с более-менее однородным составом внешнего слоя. А ветерок там нехилый, на самом деле.
Аноним 28/02/17 Втр 22:11:51  328195
>>328189
Ты это, фотки-то покажи, его ж не видно нихуя особо на его расстоянии, с чего ты взял, что он однороден?
Я гуглить uranus high res photos не хочу, гугл не то выдаст. Опять поней.
Аноним 28/02/17 Втр 22:12:26  328196
>>328183
очень разная продолжительность дня в течении года
Аноним 28/02/17 Втр 22:14:59  328198
>>328191
Хули сложного, просто полярный круг у экватора.
Аноним 28/02/17 Втр 22:17:38  328200
>>328192
Представь себе.
Voyager 1 data indicate a temperature difference between the
surface at the equator and mid-high latitudes of ∼3 K or less;
we will not know until later in the Cassini mission whether in
fact the poles are colder than the equator. The ground temperature
probably changes diurnally and seasonally (one Titan year
is 29.5 Earth years) by a few Kelvin (Tokano and Neubauer,
2002). General circulation model (GCM) results (Tokano and
Neubauer, 2002) suggest seasonal variation of air temperatures
near the surface of less than 1 K at low latitudes and up to 4 K
at the poles with a maximum temperature of ∼94 K during
the southern summer solstice, and a minimum temperature of
∼90 K during the polar winter.
Аноним 28/02/17 Втр 22:17:56  328202
>>328198
по орбите то тоже вращается
спустя 90 дней будет нормальный день
ну ты подумай, представь в голове
Аноним 28/02/17 Втр 22:22:12  328206
two-moons.jpg (22Кб, 454x249)
А я мечтаю о двух лунах на небосводе. Ну реально, чего стоило после столкновения с Землей этой массе расколоться на две части - и были бы у нас свои уютненькие Фобос и Деймос. Просто представьте, как это повлияло бы на земные цивилизации. Может, и доперли бы на заре времен, что не бывает одной-единственной верной хуйни, а их как минимум двое, а то и больше.
Аноним 28/02/17 Втр 22:23:17  328208
>>328202
Я блин в курсе, и вполне это себе представляю.
Аноним 28/02/17 Втр 22:24:34  328209
>>328206
Две большие не ужились бы. Массер и Секунда вообще невозможны, одну расколет в кольцо из-за предела Роша, а другая пиздецкие приливы вызывать будет.
Более-менее реальная конфигурация это как в кербаче, с далёким минмусом.
Аноним 28/02/17 Втр 22:24:36  328210
>>328206
Тащемта двойственность имеется в куче разных мифологий. И тройственность тоже.
Аноним 28/02/17 Втр 22:24:58  328211
>>328190
Блядь, ты тупой, или читать не умеешь? Нахуй высрался, дегенерат?
Аноним 28/02/17 Втр 22:26:18  328212
>>328208
и "полярный круг у экватора" будет за четверь года повышаться до полюса, а потом обратно
Аноним 28/02/17 Втр 22:28:02  328214
Uranusandrings.jpg (442Кб, 1494x1494)
>>328189
Потому что этот пидор холодный, как сердце твоей ЕОТ. Поэтому атмосферка у него однороднее и спокойнее. И в видимом диапазоне да, он гладкий как лысина бати, но стоит посмотреть на него в инфракрасном, то становится не все так однозначно.
Аноним 28/02/17 Втр 22:31:12  328219
1324054544764.jpg (14Кб, 293x234)
Кстати, у меня тупой вопрос - откуда сегодня такое оживление итт? Треду ещё трёх часов не исполнилось, а уже за 100 постов перевалили.
Аноним 28/02/17 Втр 22:33:04  328223
>>328219
Луна в 2018.
Аноним 28/02/17 Втр 22:34:18  328225
>>328219
Так последний день зимы. Анон начал оттаивать понемногу.
Аноним 28/02/17 Втр 22:34:30  328226
>>328219
Сам в ахуе, не успеваю всем отвечать, аж пальцы болят постить.

>>328223
Но это ещё вчера было.
Аноним 28/02/17 Втр 22:35:37  328227
>>328219
>Треду ещё трёх часов не исполнилось, а уже за 100 постов перевалили.
Это хуйня. Как-то пару лет назад перекатили тред и там кефиросрач зародился, с первых постов. За полтора часа полбамплимита насрали.
Аноним 28/02/17 Втр 22:35:46  328228
Вот вам титан https://youtu.be/hyA8KgXisFM?t=7m8s
Аноним 28/02/17 Втр 22:36:22  328229
>>328219
>Пролетариям подали новость об открытии экзопланет, как нечто грандиозное.
>У пролетариев резко появился интерес к науке, это ведь так модно)))
>Пролетарий идет в подобающий своим умственным способностям тред - тред тупых вопросов.
Не благодари.
Аноним 28/02/17 Втр 22:36:50  328230
rosetta67pfeb32[...].jpg (114Кб, 590x644)
>>328219
Приближение к перигелию, азотный снег растаял, звук в атмосфере начал передаваться.
Аноним 28/02/17 Втр 22:38:33  328231
>>328229
>практически никаких вопросов про экзопланеты в ТТВ
>специально созданный тред со взаимнометаниями говен полутора анонов за неделю до бамплимита не дорос
Неее, тут наверное не то, или не только то.
Аноним 28/02/17 Втр 22:39:17  328233
>>328230
Кстати насчёт азотного снега. Кто-нибудь следил за исследованиями атмосферы Плутона после New Horizons? Я помню что до горизонтов было 2 модели, и обе предсказывали что она выпадает в осадок в афелии. Их подтвердили или опровергли?
Аноним 28/02/17 Втр 22:40:34  328234
>>328230
Приближение к перигелию закончилось в новогодние праздники. Сейчас мы движемся к афелию.
Аноним 28/02/17 Втр 22:42:13  328237
>>328233
Ты хоть бы погуглил, когда Плутон будет в афелии.
Аноним 28/02/17 Втр 22:46:31  328241
>>328237
Что это меняет? Я о моделях которые были до Горизонтов говорю, Баруччи 2005 и Стансберри 2011. Горизонты привез новые данные, вот изменили они что-нибудь или нет. короч если ты не понял вопроса, можно на него не отвечать например
Аноним 28/02/17 Втр 22:51:01  328243
>>328241
Горизонтотред еще не утонул, если что.
Аноним 28/02/17 Втр 23:07:24  328250
>>328117
Самое смешное, что его одноклассница даже не заметит подвох.
Аноним 01/03/17 Срд 01:27:40  328267
>>328183
http://adsabs.harvard.edu/abs/1996LPI....27.1437W
Аноним 01/03/17 Срд 03:55:15  328274
>>328273
Ты не мог просто ссылку кинуть на прошлый пост, нарцисс ебаный. Обязательно надо второй раз стенкой высраться?
Аноним 01/03/17 Срд 04:10:07  328275
tuorhn.jpg (37Кб, 600x450)
>>328274
Аноним 01/03/17 Срд 13:37:54  328292
Возраст сириуса 230 млн лет, что недохуя совсем. Сириус Б прогорел и сбросил оболочку по галакт. меркам совсем недавно. Почему мы не видим туманность, тем более что она так близко?
Аноним 01/03/17 Срд 13:48:22  328294
Как выйти на орбиту кометы средних размеров?
Есть ли орбита у МКС?
Аноним 01/03/17 Срд 14:07:29  328297
>>328294
1. Легко
2.да, но сфера хилла внутри станции
Мкс на орбите кометы вполне может иметь свою сферу тяготения
Аноним 01/03/17 Срд 14:15:13  328300
Вот когда о гороскопе говорят, что Солнце, дескать, проходит по зодиакальным созвездиям, а как в древности могли наблюдать это явление? Днем же звезд не видно, затмения раз в десятки лет, ночью видно противоположную часть неба, а ведь гороскопы уже тогда были.
Аноним 01/03/17 Срд 14:34:08  328302
А если космонавт выйдет в открытый космос и ему каким-то образом удастся затормозить до скорости 0 м/с, как быстро земля притянет к себе его?
Аноним 01/03/17 Срд 14:38:00  328304
>>328302
Расстояние 400км(если с мкс)
Ускорение свободного падения 9,8(на такой высоте примерно столько же)
Этих данных достаточно
Аноним 01/03/17 Срд 14:40:00  328305
>>328304
А войти в атмосферу на такой скорости реально, чтобы не сгореть?
Аноним 01/03/17 Срд 14:41:33  328306
>>328305
нет, если плюхнуться в том виде, в котором астро/космонавты выходят
Аноним 01/03/17 Срд 14:43:23  328307
>>328301
Телескопы-то только при Галилее появились, а про эклиптику знали уже древние египтяне и майя.
Аноним 01/03/17 Срд 14:45:26  328310
>>328304
Неа, про атмосферу забыл.

>>328302
Если не сгорит каким-то образом - с суборбиталки высотой с орбиту МКС кораблю надо примерно минут 12-15 падать. Человеку в скафандре - раза в 2-3 дольше.
Аноним 01/03/17 Срд 14:45:52  328311
>>328308
Спечется тоже.
Аноним 01/03/17 Срд 14:48:41  328313
>>328308
>пару км в секунду и ещё более менее реально стерпеть
Пара км в секунду это уже добротный такой гиперзвук и нагрев градусов до 500-600 даже с баллистикой человека в скафандре, он попросту сгорит при входе в плотные слои.
Аноним 01/03/17 Срд 14:51:25  328314
>>328313
Бля жаль. А то можно было бы легко покинуть мкс.
Аноним 01/03/17 Срд 14:58:43  328316
59119083redbull.jpg (47Кб, 660x580)
63487378stfusab[...].jpg (58Кб, 1024x576)
>>328312
Даже у Юстаса и Баумгартнера при прыжке со сраных 40-41км возникали проблемы с терморегуляцией на сверхзвуке (и со стабилизацией, для чего предполагался парашют-стабилизатор в одном случае и спец. форма костюма в другом; и много с чем еще). Что там будет на 100 - мне трудно представить, но костюм со специальной аэродинамикой/противоперегрузочными мерами для спуска нужен будет точно, можно сказать наполовину спускаемый аппарат уже. В общем крайне нетривиальная это хуйня, не просто так в оба прыжка вбухали миллионы.
Аноним 01/03/17 Срд 15:01:49  328317
>>328314
Её и сейчас экстренно покинуть несложно. Всегда висят пристыкованные корабли в достаточном количестве. Пизданул метеорит - хуяк в Союз и отбыли, дело пары минут максимум. При этом на союзах можно и рядом повисеть, т.к. у них автономность на сколько-то дней.
Аноним 01/03/17 Срд 15:03:56  328318
>>328065 (OP)
> 2 пик
А что, кстати, услышит космонавт при ВКД, попытавшись сыграть на гитаре? Как исказится звук при передаче через руки и скафандр? И вообще, что и как из звуков соприкосновения слышат космонавты при работе на внешней стороне?
Аноним 01/03/17 Срд 15:20:46  328320
>>328304
>Этих данных достаточно
Нет. Во-первых, есть атмосфера, причём неравномерная. Во-вторых, в таких пределах высот уже весьма заметно изменение g.
Аноним 01/03/17 Срд 15:27:11  328323
Можно ли в космосе подрочить хуй?
Аноним 01/03/17 Срд 15:28:12  328325
>>328324
>где ближайшая чёрная дыра
Тебе подсказать?
Аноним 01/03/17 Срд 15:29:00  328326
>>328323
На космической станции? Да. Но эрекция там плоховатая.
Аноним 01/03/17 Срд 15:38:01  328329
>>328324
Ты можешь блять нормально разговаривать?

>способны ли чёрные дыры быть зародышами самых разнообразных вселенных
Нет.

>Что являет собой эта сингулярность?
Какая? Которая в дырах? Ну просто мешочек с массой, в который нельзя заглянуть.
Аноним 01/03/17 Срд 15:39:06  328330
>>328328
https://ru.wikipedia.org/wiki/%D0%A2%D0%B8%D1%85%D0%BE%D1%85%D0%BE%D0%B4%D0%BA%D0%B8
Аноним 01/03/17 Срд 15:44:50  328331
>>328326
А за бортом?)
Аноним 01/03/17 Срд 15:51:54  328333
>>328331
a) Либо ты в скафандре и не можешь дотронуться до своего заправочного шланга не снимая свитер скафандр.
/б/) Либо ты без скафандра и фап в данной ситуации тебя волнует меньше всего.
Аноним 01/03/17 Срд 15:52:11  328334
>>328331
Если ты в скафандре, специально сделанным чтоб можно было.
Аноним 01/03/17 Срд 16:00:40  328337
А где примерно черная дыра в нашей солнечной системе? Неужели не изобрели сканер черных дыр?
Аноним 01/03/17 Срд 16:50:00  328342
Чет я уже запутался. Как будет увеличиваться скорость падения человека с 400 км до слоев атмосферы?
Аноним 01/03/17 Срд 17:09:06  328343
>>328330
В космосе они перейдут в спячку.
Аноним 01/03/17 Срд 17:38:00  328344
Посоны, говорят в марте йоба метеоритные дожди должны быть. В рф их видно будет?
Аноним 01/03/17 Срд 18:06:44  328345
>>328342
Для орбитальных скоростей плотные слои начинаются на 70-80км. Для человека и суборбитальных скоростей - ну условно говоря 50-60км. считать надо в пакете вроде SCARAB хотя бы До этой отметки будет лететь с ускорением, потом очень резкое торможение с плазменной фазой и перегрузками в несколько g. На 20км и ниже устаканится до терминалки, ели будет чему устаканиваться. Поначалу сверхзвук, потом как у обычного парашютиста - в районе 200км/ч ближе к земле.
Аноним 01/03/17 Срд 18:11:30  328346
>>328117
Главное налегать на
ЧЕРНЫЕ ДЫРЫ
@
КРОТОВЫЕ НОРЫ
@
ДЕЛАТЬ ДЫРКУ


А вот про то что всего 13 только вскользь.
Аноним 01/03/17 Срд 18:36:19  328347
>>328344
>йоба метеоритные дожди должны быть
Не метеоритные дожди, а метеорные потоки, и не йоба, а так, хуйня галимая, 5-10 штук в час.
Аноним 01/03/17 Срд 18:49:59  328348
>>328339
>транзитными методами
Охуительные истории.
Их находят путем гравитационных возмущений, например по траекториям движения близких звезд и газа. А так же наблюдение аккреций на объекты, которые совершенно точно не являются нейтронными звездами. Плюс микролинзирование. Это основные способы.
Аноним 01/03/17 Срд 19:09:27  328349
Если астронавт на мкс оттолкнется от нее посильнее и полетит без страховки к земле от чего он умрет раньше, от того что сгорит в атмосфере или от того что закончится кислород?
А если оттолкнется в направлении противоположном движению станции, встретятся ли они через полтора часа или нет?
В прошлом треде было много споров о том что считать планетой, а как на счет спутников, мкс можно считать спутником? А отцепившегося от нее астронавта?
Аноним 01/03/17 Срд 19:14:43  328350
>>328332
ТС ХУЙЛО
УМРИ ОТ ПАКОСПИДВ
Аноним 01/03/17 Срд 19:18:41  328351
>>328292
> совсем недавно
давно
Аноним 01/03/17 Срд 19:19:25  328352
>>328349
> мкс можно считать спутником? А отцепившегося от нее астронавта?
Дожили блядь. Выросло поколение дебилов, которые слыхам не слыхивали про ИСЗ.
Аноним 01/03/17 Срд 19:20:58  328353
>>328352
Это понимать как да или нет?
Аноним 01/03/17 Срд 19:21:30  328354
183303 - Diavol[...].jpg (111Кб, 400x550)
1376860130576.jpg (725Кб, 650x1000)
a6d.png (578Кб, 600x600)
Я тут подумал: вот, говорят, по рассчётам, Марс должен был быть чуть ли не в 10 раз тяжелее, чем он сейчас, что, мол, моделирующие программы, имитирующие различные сценарии развития СС, указывают на это.
Отсюда возникает вопрос - за счёт чего они модулируют сценарии? Ведь вариаций - миллиарды. У нас вся СС могла состоять из Солнца и Юпитера, который бы был единственной планетой, так как вся масса всех тел СС, кроме самой звезды, была его частью. А могла бы быть 20+ планет земного типа в компактной системе уровня Кеплера или ITSATRAP!-1
А потом как-то и второй вопрос родился: а как учёные ВООБЩЕ могут определить массу того или иного объекта в космосе? Мы ведь по сути нихуя вообще ничего даже о подкорках Земли не знаем. А всё что есть - ехали манятеории через манятерии и манятеориями погоняли, но по факту именно теории же.
Аноним 01/03/17 Срд 19:22:34  328355
>>328353
Да
https://ru.wikipedia.org/wiki/Искусственный_спутник_Земли
Аноним 01/03/17 Срд 19:24:20  328356
>>328292
>Почему мы не видим туманность
В Сириусе А ее поищи. Там она.
Аноним 01/03/17 Срд 19:25:21  328357
>>328335
> А как же гипотезы в стиле: "мы живём в чёрной дыре" "раздувание пузырей пространства и времени по теории инфляционного мультиверса" "ложный вакуум"
моча > говно >>> виртуальщик
Аноним 01/03/17 Срд 19:25:24  328358
>>328349
>Если астронавт на мкс оттолкнется от нее посильнее и полетит без страховки к земле от чего он умрет раньше, от того что сгорит в атмосфере или от того что закончится кислород?
Кислород раньше кончится. Так-то он ещё кучу кругов намотать успеет вокруг Земли.

>А если оттолкнется в направлении противоположном движению станции, встретятся ли они через полтора часа или нет?
Нет конечно, ты сам вообще себе ситуацию представляешь? Если только он не оттолкнётся с первой космической скоростью, но тогда он падать начнёт. А так, будет лететь за МКС, постепенно отдаляясь от неё.

>мкс можно считать спутником?
Искусственным, да.
Аноним 01/03/17 Срд 19:30:05  328359
>>328335
>А как же гипотезы
При чём тут они? Во-первых, они все вилами по говну писаны, а во-вторых, там вселенные рождаются просто так на пустом месте в разных точках далеко друг от друга. И вообще, космологическая сингулярность и чёрные дыры — разные вещи.

>что насчёт космологической?
А ней непонятно ничего. ОТО там неприменима, а теории нормальной нет.

>Прост достал термин "Большой бабах"
Нет такого термина, есть Большой взрыв. Зачем что-то "приукрашать", тем более если всё равно красиво не получается?
Аноним 01/03/17 Срд 19:33:52  328360
>>328354
>а как учёные ВООБЩЕ могут определить массу того или иного объекта в космосе?
По гравитационному взаимодействию, по составу и размерам, по гравитационному красному смещению.
Аноним 01/03/17 Срд 19:35:10  328361
>>328166
Шланг возьми. Воду включи. И с забора на мамку струю переведи.
Аноним 01/03/17 Срд 19:35:30  328362
Можно ли на орбите собрать еба-корабль с центрифугой как в кино и уже после этого лететь на марс? Или нужно обязательно с земли стартовать?
Аноним 01/03/17 Срд 19:37:55  328363
>>328360
Раз уж это тупых вопросов тред, запращиваю пояснения для даунов обывателей попроще по каждому из озвученных тобой терминов.
Аноним 01/03/17 Срд 19:43:47  328364
>>328362
Обязательно с Земли, иначе в галактической комиссии не засчитают, скажут, так каждый дебил может, и вообще это не с Земли получается экспедиция, а откуда-то из космоса, и в учётных книжках мы так и останемся неудачниками без межпланетных полётов.
Аноним 01/03/17 Срд 19:59:16  328368
>>328363
Например, берем звезду, которая шатается, ее светимость и температуру, прикладываем к диаграмме Герцшпрунга — Рассела и узнаем, к какому типу звезд она относится. То есть теперь мы знаем примерную массу этой звезды. Потому как звезды одной светимости, цвета, спектра, обладают примерно одной и той же массой. Теперь измеряем шатание этой звезды и периодичность ее шатания, теперь можно узнать, на каком расстоянии и какой массой обладает объект, который ее шатает. Профит.

В гравитационном поле пространство искривляется. Гравитация хватает фотон за хер, от чего фотону становится неловко и он краснеет. По изменению красноты от нормальной, можно узнать, насколько охуевшая дыра там гнет пространство.
Аноним 01/03/17 Срд 20:00:47  328369
>>328363
>По гравитационному взаимодействию
Есть формула, определяющая силу притяжения, и разработана целая наука о механике гравитационного взаимодействия. Так, например, два тела могут обращаться по эллиптическим орбитам вокруг общего центра масс. Узнав параметры этого движения, легко найти их массы. Вообще говоря, это основной способ.

>по составу и размерам
Гугли плотность. Если тело состоит из вещества известной плотности, то, зная его размеры, мы можем найти его массу. Для космических тел не всё так просто, потому что под собственной тяжестью их недра сильно сжимаются, но и не всё так сложно, чтобы нельзя было это использовать. Со звёздами, например, вообще легко, у нас есть классификация звёзд (гугли Герцшпрунга-Рассела), и мы знаем, какие они бывают. Да в общем-то и с остальными объектами так.

>по гравитационному красному смещению
Согласно ОТО, в гравитационном поле замедляется время, и это приводит с изменению частоты света, если смотреть на свет, идущий к нам из гравитационного поля, будет заметно смещение его частоты к более красной.
Аноним 01/03/17 Срд 20:01:07  328370
>>328324
Сингулярностей в природе не существует.
Аноним 01/03/17 Срд 20:03:22  328371
С Альфы Центавра наше Солнце йоба-звезда уровня Сириуса или рядовая мелкая хуйня?
Аноним 01/03/17 Срд 20:04:29  328372
>>328368
Эм...
Я тебе про Фому, а ты мне про Ерёму.
Так про главную последовательность звёзд я в курсах, и там ошибочка на пару мильёньчиков километров или килограмм - так, пустяк и погрешность.
Но я-то спрашивал про параметры не звёзд хоть Арктура, хоть красного некогда Сириуса, хоть Антареса, хоть твоей мамки, а планет, будь то великан Юпитер, ни так, ни сяк Уран или какая-нибудь букашка уровня Плутона.
Аноним 01/03/17 Срд 20:06:21  328373
>>328371
Альфа центавра размером и массой примерно такоая же, как наше Солнце, поэтому Солнце с Альфы Центавра выглядит точно так же, как Альфа Центавра с Солнца.
Аноним 01/03/17 Срд 20:06:47  328374
Читал не раз, что на самом деле Туманность Андромеды занимает на небе место в несколько раз больше луны. Объясните, плез, эту хуйню. Почему в реальности её хуй увидишь? Пытался прошлой зимой, нашёл на небе Большой Квадрат, потом по приложению на мобильнике, нихуя не увидел.
Аноним 01/03/17 Срд 20:07:31  328375
>>328373
А как выглядит Альфа Центравра с Солнца? В южном полушарии не был ещё.
Аноним 01/03/17 Срд 20:08:05  328376
79f8325df92d241[...].gif (2511Кб, 500x497)
>>328371
Тогда лучше спроси, с Проксимы b наше Солнце как Сириус или нет?
Аноним 01/03/17 Срд 20:10:14  328378
hellohtmlm32950[...].gif (24Кб, 2048x1024)
>>328372
Епта, закон всемирного тяготения уже убрали из школьной программы, что ли?
Аноним 01/03/17 Срд 20:11:00  328379
>>328375
Стеллариум все еще доступен для скачивания бесплатно и без СМС.
Аноним 01/03/17 Срд 20:14:40  328382
meanwhileinaust[...].jpg (44Кб, 600x450)
>>328375
Кстати, интересно про южное полушарие. Правда ли, что там звездное небо унылое говно по сравнению с северным? Никаких ярких запоминающихся фигур вроде Ориона, Ковша, Лебедя, Квадрата, Плеяд и т. д. Стеллариум и Спейс Инжин не дают должного представления, нужно впечатление от личного опыта.
Аноним 01/03/17 Срд 20:16:03  328383
ce918e5fde98405[...].png (279Кб, 1366x768)
m31abtpmoon900.jpg (79Кб, 733x800)
>>328374
>Почему в реальности её хуй увидишь?
Обладает низкой поверхностной яркостью, из-за расстояния.

>нихуя не увидел.
Нужно съебать подальше от любых огней. Из-за светового загрязнения довольно трудно что-либо различить на небе. Невооруженным глазом можно различить мутное пятно, размером немного больше Луны.
Аноним 01/03/17 Срд 20:17:53  328384
>>328382
Да, там Луна вверх ногами, например. Довольно забавно. Зато у них можно разглядеть Магеллановы облака. Созвездия там тоже нормально складываются, просто они для тебя непривычны, глазу зацепиться не за что. С непривычки.
Аноним 01/03/17 Срд 20:27:14  328385
>>328384
В какой момент она переворачивается? Если плыть по меридиану на йух?
Аноним 01/03/17 Срд 20:28:09  328386
>>328321
Большая часть взрыва сверхновой как раз не в видимом спектре. Так что если она будет светить так, что будет видна очень ярко, то и фонить будет осне круто.
>подогрев Земли на несколько градусов\десятков градусов в течении недель\месяцов пока это второе Солнце будет собственно светить
this kills the life on earth
Хорошенько так стерилизует причём. Может разве в океанах на днище ёбаном и останутся какие бактерии.
Аноним 01/03/17 Срд 20:36:08  328392
>>328219
Популяризаторы от науки. Ютабы завалены видяхами с самым разнообразным матаном. Некоторый оказывается слишком сложным и шкальники после посещения фаптреда заходят узнать что будет если поссать в касмасе и тип пачиму скорость светы конечна по ото и что блядь ваще такое ото.
Аноним 01/03/17 Срд 20:36:36  328393
Moon-observer.jpg (555Кб, 1650x1275)
>>328385
Ни в какой момент, это не Луна поворачивается, а ты смотришь на нее под другим углом. Ближе к экватору месяц вообще рогами вверх светит.
Аноним 01/03/17 Срд 20:45:05  328397
>>328393
А если идти в другую сторону (не по 0 а по 180 меридиану) то будет вниз рогами, выходит? Но это ж фигня выходит, днём вверх рогами, ночью вниз рогами (Земля ж повернулась). или Луну вместе с ней мотыляет?
Аноним 01/03/17 Срд 20:54:01  328399
>>328382
Но там няшный южный крест
Аноним 01/03/17 Срд 20:55:29  328401
>>328397
Направление рогов зависит от широты, а не долготы, дятел.
Аноним 01/03/17 Срд 20:56:33  328402
>>328382
Оттуда альфу центавра видно, это вообще самое близкое что есть.
Аноним 01/03/17 Срд 20:56:59  328403
devil-horns-in-[...].jpg (15Кб, 500x706)
>>328393
А ещё где-то там такое бывает во время затмения.
Аноним 01/03/17 Срд 21:02:13  328407
>>328403
Сотона рождается на фоне божьего храма свят свят свят
Аноним 01/03/17 Срд 22:00:26  328431
>>328374
>Почему тусклый объект не видно?
Ну хуй знает. Наверное, потому, что он тусклый?
Аноним 01/03/17 Срд 22:05:02  328433
Поясните за Кесслера. Допустим, на орбиту запустили много всего хлама и оно там начало сталкиваться друг с другом. В результате количество объектов растёт, но их суммарный вес - нет. Значит средний размер объекта уменьшается. И этот процесс необратим. Значит средний размер будет рано или поздно очень-очень маленьким и это приведёт к образованию колец вокруг Земли. Как у Сатурна. Или там какие-то другие кольца, более культурные, не подверженные Кесслеру?
Аноним 01/03/17 Срд 22:08:23  328435
>>328431
Но он не тусклый, есть гипотеза что он ярче млечного пути т.к. звёзд больше. Или там обратная зависимость, типа весь свет зажирает в себя смчд в центре.
Аноним 01/03/17 Срд 22:09:39  328437
>>328435
Она, блядь, дохуя далеко, в отличие от млечного пути, и тем паче луняши. А никакая не смчд. Звезд больше, да, но наши ближе.
Аноним 01/03/17 Срд 22:16:43  328441
p0031.png (91Кб, 650x1048)
p0032.png (141Кб, 650x1048)
>>328372
>Но я-то спрашивал про параметры не звёзд
>а планет, будь то великан Юпитер, ни так, ни сяк Уран или какая-нибудь букашка уровня Плутона.
Учебник астрономии — ответит на большинство ваших вопросов в этом ИТТ треде! Акция — 100% скидка на скачивание из бесплатной онлайн-библиотеки!
Аноним 01/03/17 Срд 22:17:46  328442
>>328402
Самое близкое что есть — это луна, её отовсюду видно.
Аноним 01/03/17 Срд 22:36:10  328449
>>328437
В смысле далеко? Это самая близкая голактеко вообще-то. И ещё и летит, зараза, прямо в нас.
Аноним 01/03/17 Срд 22:37:37  328452
>>328091
>International Docking System Standard
Гомосячество какое-то
Аноним 01/03/17 Срд 22:38:47  328454
>>328449
>В смысле далеко?
В смысле 2 500 000 световых лет - это тебе не хуй собачий.
Аноним 01/03/17 Срд 22:40:30  328456
>>328119
>огромных ножниц
Гипотетических нематериальных ножниц
Аноним 01/03/17 Срд 22:40:47  328457
Насколько близко могут находиться гравитационно не связанные звёзды?
Аноним 01/03/17 Срд 22:41:13  328458
>>328452
То ли дело гетеросексуалный штырь-конус.
Аноним 01/03/17 Срд 22:41:33  328459
>>328225
>Анон начал оттаивать понемногу
У шизофреников повышается активность
Пофиксил тебя
Аноним 01/03/17 Срд 22:43:40  328461
>>328459
не каркай, ещё тьфу-тьфу-тьфу не просыпались
Аноним 01/03/17 Срд 22:46:45  328463
>>328457
Настолько, насколько влияние других объектов, или объекта будут влиять на эти звезды, сводя взаимное гравитационное влияние к пренебрежимо малому минимуму.


>>328461
>>328459
А нихуя, уже неделю как обострение у бабки с соседнего дома. Она по весне затыкает все дырки в раковинах и ванной и открывает краны, чисто по фану топит соседей снизу. Вся эта хуйня продолжается, пока ее в дурдом не упекут. Она там полежит, обострение пройдет и домой отпускают, на следующую весну все по-новой.
Аноним 01/03/17 Срд 22:47:55  328464
>>328449
>Это самая близкая голактеко вообще-то
https://ru.wikipedia.org/wiki/Карликовая_эллиптическая_галактика_в_Стрельце
https://ru.wikipedia.org/wiki/Карликовая_галактика_в_Большом_Псе
Аноним 01/03/17 Срд 22:49:30  328465
>>328463
Цифру можешь сказать. Для таких звёзд с характеристиками как Солнце или для красных карликов. Насколько ближе должна быть центавра чтобы система стала гравитационно связанной?
Аноним 01/03/17 Срд 22:56:36  328467
>>328457
дело не в расстоянии, а в относительной скорости
Аноним 01/03/17 Срд 22:58:28  328468
>>328467
Скорость и все остальные нужные параметры бери как у Солнца. 200-250 км/с или сколько там.
Аноним 01/03/17 Срд 23:00:01  328469
hellohtmlm32950[...].gif (24Кб, 2048x1024)
>>328465
Нет, не могу, если ты спрашиваешь цифру, значит ты нихуя не понял, перечитывай до полного просветления.
Гравитационное взаимодействие простирается бесконечно далеко, просто оно ослабевает с квадратом расстояния. Если гипотетически разнести две одинаковых звезды на миллиард световых лет, в ебаном сферическом абсолютном нихуя, то они будут покоится относительно друг-друга миллиард лет, пока силы взаимного гравитационного воздействия не достигнут их. После этого эти две звезды начнут сближаться, притягиваясь все сильнее и сильнее.
Центавра имеет на наше Солнце гравитационное влияние, просто оно не такое сильное, что бы преодалеть все прочие влияние других объектов, или как-то повлиять на относительные скорости. По сути изначально твой вопрос некорректен.
И вообще я не понимаю, откуда за вечер взялся второй дебил, который не знает о формуле всемирного тяготения

Аноним 01/03/17 Срд 23:03:15  328471
>>328469
Тебе же сказали, бери конкретные числа, конкретные массы, конкретные скорости. Другие объекты - звёзды в Млечном пути же. Ихнее количество, массы, размеры, скорости - всё известные величины. Ну хорошо, влиянием гравитации иных галактик можешь пренебречь. Можешь не пренебрегать.
Аноним 01/03/17 Срд 23:03:39  328472
>>328468
> относительной
значение знаешь?
Аноним 01/03/17 Срд 23:05:25  328474
>>328471

Иди-ка ты нахуй. Сам считай, миллиарды параметров. Формулу я тебе предоставил.
Аноним 01/03/17 Срд 23:05:42  328475
>>328472
Лол, по ходу ты не знаешь.
Хорошо, подскажу, умножай на два. Солнце летит 200км/с, в сторону Солнца летит объект с Проксиму с такой же скоростью. Скорость сближения 400 км/с значит. Насколько близко она должна пролететь, чтобы не разлетелись, а потом начали крутиться друг вокруг друга.
Аноним 01/03/17 Срд 23:07:05  328477
>>328474
Какие миллиарды параметров? Бери один - средне усреднённый. Короче, слился - так и скажи.
Аноним 01/03/17 Срд 23:08:27  328478
14391634064210.gif (1703Кб, 200x150)
>>328475
>>328477
Сука, до чего забавный зверек.
Аноним 01/03/17 Срд 23:09:03  328480
>>328475
хуйню несёшь
Аноним 01/03/17 Срд 23:17:45  328489
0d9a75112ec5b6o[...].jpg (607Кб, 1300x869)
Котаны, объясните городскому дауну, который такой штуковины на небе ни разу не видел за свою молодую жизнь, хочу летом съебаться в карелию, увижу ли я не природе, далеко за городом в лесу похожую картину? Это ж блять мега грандиозно, у меня прям очко сжимается от такой монументальной хуевины, застывшей на всем небе по горизонту.
Аноним 01/03/17 Срд 23:19:26  328491
>>328489
это фотачка с оче большой выдержкой
но ты всё равно охуеешь, базарю
Аноним 01/03/17 Срд 23:22:41  328492
>>328489
Такое не увидишь. Но вообще россыпь охуенная на небе, очень впечатляюще все это выглядит.

Летом в Карелии тем более ты соснешь, там белые ночи, лол. Темнота наступает не полная, и на пару часов всего. Так что фейл.
Аноним 01/03/17 Срд 23:22:49  328493
IMG9538.jpg (256Кб, 1000x1500)
>>328489
Прям такого ты не увидишь, будет что-то вроде пикрилейтед (только в живую это выглядит намного охуеннее и отчётливее). Но как этот и говорил >>328491, всё равно охуеешь.
Аноним 01/03/17 Срд 23:38:58  328503
>>328383
Хрень слева в видимом спектре?
Аноним 01/03/17 Срд 23:40:06  328504
>>328503
Да.
Аноним 01/03/17 Срд 23:43:07  328505
>>328477
Чтобы сохранялась гравитационная связанность, относительная скорость сближения (а потом и удаления) должна быть меньше третьей космической скорости для потенциально образованной системы.

Эта третья космическая будет зависеть от массы новой системы и предельного (минимального) расстояния между ними. Например для объекта, находящегося от Солнца на расстоянии Земли 16,650 км/с при условии что масса пренебрежима мала (как у Земли она мала относительно Солнца). Но это с учётом орбитальной скорости (30), итого получаем 46,9 км/с.

Данные отсюда.

https://ru.wikipedia.org/wiki/%D0%A2%D1%80%D0%B5%D1%82%D1%8C%D1%8F_%D0%BA%D0%BE%D1%81%D0%BC%D0%B8%D1%87%D0%B5%D1%81%D0%BA%D0%B0%D1%8F_%D1%81%D0%BA%D0%BE%D1%80%D0%BE%D1%81%D1%82%D1%8C


Если же это звезда размером с Солнце, то эта скорость должна быть существенно выше для разлёта, т.к. не только Солнце притягивает объект, но и объект притягивает Солнце. Формулу уже кидали.

В действительности относительные скорости звёзд далеко не 400 км/с, этот предел достигался бы если бы звезда летела в галактике "в обратную сторону". В действительности орбиты направлены в одну и ту же сторону, а относительные скорости не превышают десятков км/с, а порой ещё меньше.

Основная проблема вот в чём. Если скорость взаимного пролёта будет немного выше этой третьей космической, то две звезды окажут друг на друга сильное гравитационное влияние. Они будут притягиваться не разово в момент сближения, но и до и после, а значит гравитационные силы постоянно будут "тормозить" их друг относительно друга.

На самом деле чтобы две звезды стали гравитационно связанными, надо пролететь очень-очень близко и лететь примерно в одном направлении (тогда относительная скорость будет ниже). Даже если условный красный карлик пройдёт в 0.1 светового года со скоростью 1 кс/с, этого явно будет мало, правда Солнечную систему при этом нехило распидорасит. Кучу объектов выкинет из неё нахуй. Какие-то объекты, наоборот будут захвачены из системы той звезды. Получим дохуя новых долгопериодических комет и все такие удовольствия.

Как видим, чтобы произошёл гравитационный захват, звезда должна быть или очень массивной, или пройти буквально в районе пояса Койпера, счёт ведётся не на световые годы, а на считанные астрономические единицы. Но бывает это крайне редко. У кратных звёзд и скоплений совсем другие механизмы формирования. Это не одна звезда, которая прошла рядом с другой и стали крутиться вместе. Это обычно именно что формирование в одной системе нескольких объектов звёздных масс. Вот даже у нас Юпитеру до звезды не так уж и много массы не хватило. Будь он в 20 раз тяжелее, был бы коричневым карликом, а в 100 раз тяжелее - был бы красным карликом и у нас была бы полноценная двойная система.

При этом расстояние звёздными между объектами в двойной системе бывают меньше 1 а.е., а иногда достигают тысяч а.е.

Фуф, думаю что ответил.
Аноним 01/03/17 Срд 23:44:12  328506
>>328504
И фото делают с хабла?
Аноним 01/03/17 Срд 23:45:38  328507
>>328506
Можно и с обычного телескопа в ебенях.
Аноним 01/03/17 Срд 23:46:07  328508
>>328489
А что это светит на горизонте?
Если восход/закат, то явно было бы светлее.
Аноним 01/03/17 Срд 23:47:46  328509
>>328508
Очевидные огни города на большой выдержке.
Аноним 01/03/17 Срд 23:54:55  328512
Что за херня сегодня весь вечер висела в небе? Вроде как звезда, ибо постоянно на месте стояла, но по яркости как месяц который как бы "слева" от этой звезды был, серьезно, по сравнению с полярной звездой намного ярче и больше
Аноним 01/03/17 Срд 23:55:35  328513
>>328512
венера
Аноним 01/03/17 Срд 23:57:43  328514
>>328513
А не, я уже нашел, взрыв сверхновой

Поразительно яркие ударные волны, исходящие от взрыва массивной сверхновой звезды SN 1987A, впервые замеченные с Земли в 1987 году, можно наблюдать и сегодня, спустя три десятилетия.

Взрыв этой сверхновой звезды произошел всего в 160 000 световых лет от Земли в спутниковой галактике Млечного Пути — Большом Магеллановом Облаке.
Аноним 02/03/17 Чтв 00:00:05  328515
>>328514
Вообще думаю многие заметили ебалу, которая светила на ровне с месяцем если не ярче лол, и была видна когда даже других звезд не было видно
Аноним 02/03/17 Чтв 00:01:52  328516
MyCn18-HST.jpg (59Кб, 549x375)
>>328514
Ты ебанулся, их только в Хаббл какой-нибудь можно наблюдать. Обчитаются своих википедий и ябуть друг друга в жопы.
Та хуйня - Венера, она сейчас близко к нам и очень яркая, поэтому это уже платиновый вопрос пейсача.
Аноним 02/03/17 Чтв 00:02:39  328517
blob (350Кб, 562x434)
>>328514
Аноним 02/03/17 Чтв 00:05:34  328519
blob (3894Кб, 1824x1195)
>>328506
Вот чувак буквально на тапок снял. По меркам любительской астрономии сетап у него ниже среднего
http://www.astrobin.com/261824/B/
Аноним 02/03/17 Чтв 00:09:34  328522
>>328516
Ну лан тогда. А на каком кстати расстоянии должна въебать сверхновая что бы не вооруженным взглядом, это выглядело как просто очень яркая звезда?
Аноним 02/03/17 Чтв 00:11:57  328524
>>328522
В нашей галактике.
Эта, например, 6850 световых лет.
https://ru.wikipedia.org/wiki/SN_1006
Аноним 02/03/17 Чтв 00:12:22  328525
>>328522
Данная сверхновая наблюдалась лишь год-два и только (ну, почти) в Южном полушарии. Вспыхнула бы сверхновая в нашей галактике - все бы трубили. На каком расстоянии? Она даже будет видна с галактики Андромеды, но на пределе, около 6m
Аноним 02/03/17 Чтв 00:16:11  328527
1487775646.jpg (111Кб, 960x960)
>>328519
>сетап ниже среднего
>NEQ6 PRO
Аноним 02/03/17 Чтв 00:18:20  328528
На каком минимальном расстоянии сверхновая вообще может вспыхнуть? Тут писали про 6850, ближе можно? Насколько, где предел? И как это будет выглядеть (звёздная величина и т.д.)?
Аноним 02/03/17 Чтв 00:19:22  328529
>>328519
>на тапок снял
Грамотное использование тапка прямыми руками: легкий 6" рефлектор, монтировка с двукратным запасом по весу, следящиий привод от дополнительной камеры и явно все хорошо отрегулировано.
Аноним 02/03/17 Чтв 00:20:08  328531
>>328528
Вот, Бетельгейзе в конце своей жизни вспыхнет как сверхновая, а она в 650 световых годах от нас. Звёздная величина будет около -10m, то есть почти как вторая Луна, только звёздочка. Будет отлично видна днём, если даже -4,5m видны днём
Аноним 02/03/17 Чтв 00:21:39  328532
>>328531
> а она в 650 световых годах от нас
это сейчас
Аноним 02/03/17 Чтв 00:22:48  328533
>>328532
Так всё равно нет ближнего предела вспышки сверхновой к Земле
Аноним 02/03/17 Чтв 00:24:03  328534
>>328533
вот так сразу бы
Аноним 02/03/17 Чтв 00:27:28  328535
>>328512
>по сравнению с полярной звездой
Есть ещё те люди, которые думают, что Полярная очень яркая. На самом деле она тусклая
Аноним 02/03/17 Чтв 00:29:59  328536
>>328535
самая известная и просто найти
в связи с этим логично что ньюфаня использовал её как эталон
Аноним 02/03/17 Чтв 00:32:22  328537
>>328527
Все остальное ты проигнорировал?
Аноним 02/03/17 Чтв 00:42:15  328540
48mgjs.jpg (17Кб, 600x348)
>>328493
то есть газо-пылевых облаков вообще не увижу чтоле, только широченную дорогу скоплений звезд или все таки чуть видно будет? И блядь неужели человек не способен уведить такое даже в открытом космосе? Какой же хуевы глаз у лысой обезьяны
Аноним 02/03/17 Чтв 00:46:34  328542
>>328533
С хуя ли? Она есть, по крайней мере на миллионы лет. Определяется массой ближайших звезд, и их возрастом. Перебрав всю ближайшуйю пиздобратию на ближайшие несколько десятков миллионов лет можно успокоится.
Аноним 02/03/17 Чтв 00:48:45  328543
>>328540
>то есть газо-пылевых облаков вообще не увижу чтоле
Не увидишь.
>только широченную дорогу скоплений звезд
Да.
>И блядь неужели человек не способен уведить такое даже в открытом космосе?
Неспособен.
>Какой же хуевы глаз у лысой обезьяны
Увы.
Аноним 02/03/17 Чтв 00:50:53  328544
>>328537
А что тебе еще нужно? 6" вполне достаточно для астрофоточек, учитывая что с такой монтировкой и гидом можно без труда в длинные выдержки.
Аноним 02/03/17 Чтв 00:54:10  328545
>>328544
>А что тебе еще нужно?
Скилл на этом сетапе, получить такой результат. Учитывая бюджетность трубы и камеры, все это не так тривиально, как тебе кажется.
Аноним 02/03/17 Чтв 01:03:16  328546
>>328545
Скилл там скорее в постобработке, нежели в получении самих равок. Камера далеко не йоба, но на исо200 даже она особо шуметь не будет + электрофокусер и маска Бахтинова позволит без труда выдрочить резкость.
Аноним 02/03/17 Чтв 01:08:38  328548
>>328546
Диванного специалиста видно издалека.
Аноним 02/03/17 Чтв 01:13:20  328549
>>328548
>кукареку
А мог бы и в аргументацию.
Аноним 02/03/17 Чтв 02:47:34  328562
>>328561
> обоснование
ой всё
Аноним 02/03/17 Чтв 06:47:51  328573
>>328332
Смотри. Ты пытаешься объяснить нечто предельное в рамках теорий, границы применимости которых не распространяются на этот случай. В результате у тебя происходит деление на нуль. Разделить на нуль возможно, гугли теорию колёс не тех, о которых ты подумал но не в рамках арифметики.

У каждой теории и модели есть границы применимости. Ньютоновская механика работает только при скоростях много ниже скоростей света. Что за силы начинают действовать на планковских масштабах (планковская энергия, температура, длина, масса) мы понятия не имеем. А ты хочешь чтобы тебе это объяснили через какую-то гравитацию. Через неё даже электромагнетизм не объяснишь. Нужна новая теория, теория чёрных дыр, сингулярности, вот этого всего. Может быть там будет эээ... доминировать блядь, опять это слово, везде оно какое-то другое взаимодействие, которое пренебрежимо мало в нормальном мире. Мы пытаемся создать теорию всего, предполагая что есть 4 основных взаимодействия, а в действительности их может быть больше. Только некоторые настолько незначительны что мы их никак не можем зафиксировать. Потому что чтобы они начали превозмогать, надо сжать всё вещество в космосе до размеров твоего Хуя.

И в результате там окажется что эта самая сила способна на таких энергиях создавать вещество, а с ним и свойства вещества - энергию, расстояние, время и т.д., а затем уже и барионы из чего-то такого потенциального. По механизму, чем-то напоминающему рождение пар и аннигиляцию. Там происходит некий фазовый переход от запланковских величин к тому что мы называем реальным миром. Все существующие теории там просто не работают, показывая деление на нуль. Так же как не будет работать евклидова геометрия на сфере. Сумма углов треугольника больше 180. Всё, пиздец. Там просто происходит фазовый переход, наподобие замерзания воды. Она превратилась в лёд и физика жидкостей на это вещество больше не действует. Куда делось вещество, на которое действует физика жидкостей - а нет его больше.

Так вот, можно предполагать что там, в этой космологической сингулярности всякие законы сохранения перестают работать, возможно создание вещества из нихуя и прочие прелести жизни. И это нормально. Точно так же как нормально гравитационное замедление времени, хотя уже это выглядит адовым пиздецом. Ничего, привыкли к мысли что так бывает - привыкнем когда-нибудь и к новому взгляду на законы сохранения, жизнь, вселенную и всё такое.
Аноним 02/03/17 Чтв 07:02:52  328575
>>328531
>-4,5m видны днём
Это что за объекты с такой светимостью? Венера даже меньше.
Аноним 02/03/17 Чтв 07:30:57  328577
>>328575
> Венера даже меньше.
в 2019-2020 у меня какраз
Аноним 02/03/17 Чтв 07:33:27  328578
>>328388
>>328565
Аноним 02/03/17 Чтв 09:56:00  328599
>>328065 (OP)
Спейсаны, как поставить преверженца плоской земли на место?

Исходя из опыта споров, самый действенный и неоспоримый аргумент - это тень, которая затмевает луну полукругом, а не ровной тенью.

Какие еще есть доказательства, которые хуй оспоришь?
Аноним 02/03/17 Чтв 10:04:27  328600
>>328599
>Спейсаны, как поставить преверженца плоской земли на место?
Попроси, чтобы он тебя свозил на край земли.
Аноним 02/03/17 Чтв 10:42:27  328613
>>328599
>Спейсаны, как поставить преверженца плоской земли на место?
Перестать общаться с подобными шизофрениками?
Аноним 02/03/17 Чтв 11:40:14  328616
>>328575
>что за объекты с такой светимостью?
Венера, она сейчас -4,6m
Аноним 02/03/17 Чтв 12:33:05  328621
>>328599
Да где вы их блять находите?
Никогда не встречал в жизни.

Что они говорят, если показать планету с геостационарки?

ну и стопроцентный вариант показать, как корабль уходит на горизонте
Аноним 02/03/17 Чтв 12:49:11  328623
>>328621
За день на политаче как минимум 1 тред. /zog/, в который я даже не захожу, там хуями обложат и даже дискутировать не будут.
Ну и вот тебе тогда. https://vk.com/flatearthsociety

Раньше я думал, что в группе только троли, но оказалось что нет.

>Что они говорят, если показать планету с геостационарки?

Говорят, что спутников не существует лол. Кто что говорит.

>>328613
Я не общаюсь. Цели две: просвещение и лулзы. Захожу во всякие группки и вбрасываю факты и аргументы в пользу шарообразной.
Аноним 02/03/17 Чтв 12:50:40  328624
>>328621
Вот кстати, что мне ответили в сообществе на "луну".

>возьми лист бумаги и посвети сзади. Потом расскажи как тень стала одномерной.
Аноним 02/03/17 Чтв 13:42:40  328630
183224.image0.jpg (44Кб, 535x364)
Почему говорят что "время" на корабле должно замедлиться? Близнецы, вот это всё. Почему оно вообще должно идти в двух разных местах как-то иначе? Может, луч лазера при скорости корабля близкой к световой и должен замедлиться(не сама скорость света, а дойдет до приемника медленнее, чем при низкой скорости, с задержкой) - и для постороннего наблюдателя, и для пассажира судна(?) Но не само же... биологическое время людей, допустим.
В том же советском фильме в примере с вагоном поезда существуют два вагона: один пассажирский, другой - наблюдателя и двери для обоих открываются не одновременно.
Не могу понять вот этого логического прыжка "типа, хуе-мое, относительно, хуяк! - время". Оптические эффекты всякие - это я могу понять, но время...
Аноним 02/03/17 Чтв 13:49:34  328633
Наплыв шизойдов в раздел. На них весна так влияет?
Аноним 02/03/17 Чтв 14:06:11  328639
>>328630
Замедляется именно само время, и биологическое время людей, и время вообще. Иначе не получится никаких преобразований систем отсчёта с сохранением скорости света, остюда и берутся преобразования Лоренца, и замедление времени. И это давным давно доказанные на опыте вещи.
Аноним 02/03/17 Чтв 14:12:26  328641
>>328640

>>328631 (OP)
>>328601 (OP)
>>328604

Аноним 02/03/17 Чтв 14:17:28  328642
>>328599
Это совершенно бесполезно, можешь не пробовать. Если бы это было просто заблуждением, то его развеять было бы очень просто, а так, они намеренно придумали себе эту хуйню просто чтобы можно было указывать всем, что они неправы, и воображать себя этакими знатоками, ловко разрушающими основы основ и привычные картины мира.
Аноним 02/03/17 Чтв 14:31:52  328645
>>328273
Два чаю господину
Аноним 02/03/17 Чтв 16:09:54  328651
>>328599
Звёздное небо. Какого хуя в южном полушарии мы видим одни звезды, а северном другие.
Аноним 02/03/17 Чтв 16:18:16  328652
Народ, если начать падать прямо у верхней границы атмосферы, то будет ли увеличиваться моя скорость по идее трение должно слегка уменьшать ускорение или не будет никакого трения из-за объекта малого объема?
Аноним 02/03/17 Чтв 16:29:06  328653
>>328652
Посмотри данные по стратосферному прыжку Баумгартнера.
https://www.youtube.com/watch?v=ZyIVaZXDhho
Скорость твего падения соответствовать закону Бернулли. Сначала она будет расти (но чуть медленнее чем в вакууме), а потом устаканится на уровне терминальной скорости.
Аноним 02/03/17 Чтв 16:32:24  328654
В теории можно выйти на орбиту на высоте 200-300км?
Аноним 02/03/17 Чтв 16:37:39  328655
>>328654
А что, собственно, должно этому помешать? На таких орбитах дохуя чего висело, только там торможение об атмосферу быстрое.
Аноним 02/03/17 Чтв 16:38:43  328656
>>328655
Например что? И есть ли формула для расчета скорости выхода на орбиту для разных высот?
Аноним 02/03/17 Чтв 16:43:15  328657
>>328656
Например Востоки и Восходы, у некоторых перигей до 160 км опускался.
Аноним 02/03/17 Чтв 17:26:17  328658
>>328654
В теории можно выйти на орбиту на уровне моря. И даже на глубине Марианской впадины. Правда недолго.
Аноним 02/03/17 Чтв 17:51:26  328662
>>328654
Опорная орбита многих КА и то ниже. Долго на ней не задерживаются, впрочем, уходят на высокую. Ниже 250км затруднено длительное существование и требуется аэродинамическая форма как у УС-А или GOCE, ниже 160км аппарат может тупо зарыться в атмосферу и сойти.

Самые низкие рабочие орбиты были наверно у плёночных разведывательных КА, которым надо было крутануть несколько витков и спустить плёнку, и которые запускали пачками по необходимости. У Keyhole 7 бывали перигеи ниже 100км, если не ошибаюсь.
Аноним 02/03/17 Чтв 20:34:17  328675
KH7GambitDispla[...].jpg (405Кб, 1950x1295)
>>328662
> Keyhole 7
Спасибо за наводку, я раньше думал, что с перигеем ниже 150 км КА вообще даже одного витка не сделает.
А оказывается, что эта няшка почти 5 суток продержалась.
>NSSDCA ID: 1965-019A
>Launch Date: 1965-03-12 at 19:26:00 UTC
>Decay Date: 1965-03-17
>Periapsis 93 km
>Apoapsis 155 km

Аноним 02/03/17 Чтв 20:51:16  328677
tumblrinlineo87[...].jpg (69Кб, 500x667)
>>328397
рога меняются на противоположное когда луна растёт или стареет
Аноним 02/03/17 Чтв 20:55:06  328678
>>328599
Флайтрадар, возьми несколько рейсов, например с одинаковым временем полета и расстоянием, но разными наклонениями. Желательно какой-нибудь через полюс рейс и какой-нибудь по экватору.
Аноним 02/03/17 Чтв 20:57:00  328679
>>328639
>само время
Время - мера процессов.
>>328630
Поэтому см. на эту картинку. Тут показано, почему луч лазера проходит больший путь. Тот же больший путь проходят фотоны связывающие твоё тело э/м взаимодействием, то же с тремя другими взаимодействиями.
Аноним 02/03/17 Чтв 20:58:51  328680
>>328651
Не желающий сдаваться плоскоземельный даун в итоге придумает математику с искривлением геометрии, чтобы это >>328651 и это >>328678 объяснить, в итоге будет ебическая правильная математика, описывающая обычную шарообразную землю.
Аноним 02/03/17 Чтв 20:58:55  328681
>>328678
Поотыгрываю упоротого плоскоземельца
Врёти! Жидомасоны из руководства флайтрадара специально искажают отображаемые траектории.
Аноним 02/03/17 Чтв 21:00:18  328682
>>328681
Возьми сам и полетай. Спроси летавших людей.
Аноним 02/03/17 Чтв 21:04:26  328683
>>328682
Летавшие люди не знают своих истинных путевых координат, а если знают, то получают их через GPS, которая, конечно же, является грандиозной мистификацией, так как на самом деле сигналы транслируются скрытой сетью стационарных наземных башен-излучателей.
Аноним 02/03/17 Чтв 21:06:48  328684
>>328683
Большую часть маршрутов можно пройти землей и морем измеряя расстояние самолично, так, собственно, раньше и делали, и получили те самые карты которые мы имеем сейчас.
Расстояние от Лондона до Калькутты до Пекина до Москвы особо не подделаешь, чтобы миллионы путешественников не обеспокоились.
Аноним 02/03/17 Чтв 21:10:42  328685
>>328684
Ты расстояние от Лондона до Калькутты лично рулеткой измерял? Откуда ты знаешь, что числа, написанные в Википедии — правильные? А насчет карт аргумент вообще смехотворен, всем думающим людям известно, что они составляются под надзором жидомасонов, которые всех, кто пытаются им возражать и открыть людям глаза на истинное положение, объявляют психами и гноят в психушках.
Аноним 02/03/17 Чтв 21:16:52  328686
Ну ебана, весеннее обострение.
Аноним 02/03/17 Чтв 21:18:42  328687
>>328685
ОЙ, ВСЁ.
Ухожу.
Я в реале имел подобный спор, вроде аргументированный, но его аргументы ещё хоуже твоих. Еле сдержался и не переебал.

>>328599
Кстати, а какие их пруфы плоской земли?
Сейчас объективный факт, что земля не плоская. Они делают экстравагантное читай: ебанутое утверждение, что она плоская. Бремя доказательств на утверждающем. Какие у них пруфы кроме двухмерности их мозга не могущего в третье измерение?
Аноним 02/03/17 Чтв 21:50:13  328689
0[1].jpg (29Кб, 480x360)
>>328687
>ОЙ, ВСЁ.
>Ухожу.
КАК ВИДИТЕ, ВЫПУКЛОБЛЯДИ В ЭТОМ ИТТ ТРЕДЕ НЕ СМОГЛИ ПОДОБРАТЬ НИ ОДНОГО ДОСТОЙНОГО АРГУМЕНТА И ПРЕДСКАЗУЕМО СОСНУЛИ МОГУЧЕГО ПЛОСКОГО Х-ЦА
Модер, шучу, не забань еще ненароком.
Я зато ИРЛ по работе не один раз сталкивался со спид-диссидентами. Аргументация у них такого же уровня. Общий вывод от общения с подобными личностями — если человек идиот, то это надолго. Некоторых из глубин манямирка не может вывести даже гибель собственных детей, вызванная отказом давать им лекарства. ИЧСХ, они при этом еще и умудряются подавать в суд на врачей-вредителей, обвиняя их во всех своих проблемах со здоровьем. На этом фоне секта плоскоземельцев — просто очередные безобидные чудаки. По крайней мере, их идиотизм не приводит к мучительной смерти и заражению других людей.
Аноним 02/03/17 Чтв 22:05:08  328691
>>328675
Ниже уровня кармана перигелий?
Аноним 02/03/17 Чтв 22:07:06  328692
>>328691
>перигелий
>уровень Кармана
Это всё равно, что говорить об ареосинхронной орбите луны.
Да, перигей ниже линии Кармана.
Аноним 02/03/17 Чтв 22:13:40  328693
>>328691
Так у американцев всё в милях, поэтому атмосфера раньше заканчивается.
Аноним 02/03/17 Чтв 22:23:11  328695
G-PR-015-2b.jpg (51Кб, 960x720)
>>328693
Тралируешь небось.
Аноним 02/03/17 Чтв 22:55:44  328696
>>328531
Этот конец жизни когда? Через 100000 лет? Или примерно через 1000? Как там определяют когда звезда ещё пару лет протянет, а когда уже совсем наладом дышит?
Аноним 02/03/17 Чтв 23:04:41  328697
>>328675
Там просто эллиптическая, а не круговая. И аэродинамическая форма КА (УС-А/АМ например тоже были обтекаемые)
Аноним 02/03/17 Чтв 23:13:58  328699
411608.jpg (2719Кб, 2560x1600)
Вот есть у нас самолёт SR-71, мы его разогнали до 3.3 маха, или сколько там он летал. Это примерно 1/7 орбитальной скорости, т.е. ему было проще летать за счет уменьшенного веса, верно?

А если мы запилим самолёт, который будет летать строго на орбитальной скорости и не будет нуждаться в крыльях, это будет еще самолёт или уже ИСЗ?
Аноним 02/03/17 Чтв 23:15:50  328700
>>328696
>наладом дышит
Лол, каким ещё наладом? На ладан, это благовоние церковное.

Никак там не определяют, очень размытые границы, она в конце жизни по сравнению со сроками жизни звёзд, вот и суди сам. Там и миллион лет — это так, фигня.
Аноним 02/03/17 Чтв 23:18:11  328701
>>328699
Прочитай определение Линии Кармана.
Аноним 02/03/17 Чтв 23:18:30  328702
>>328699
Аноним 02/03/17 Чтв 23:18:59  328703
foto-zloy-odess[...].jpg (48Кб, 702x469)
>>328696
>наладом дышит
На ладан, сын мой, на ладан.
Раньше перед смертью обычно старались старались покаяться в грехах, причаститься и собороваться, откуда и пошло выражение.
Аноним 02/03/17 Чтв 23:24:28  328705
>>328699
Самолет по определению летает за счет подьемной силы крыльев. У тебя не самолет.

>>328703
>>328700
Иная старуха бывает про руха. Нельзя же все фразеологизмы знать.
Аноним 02/03/17 Чтв 23:26:48  328706
>>328701
>примерно на этой высоте атмосфера становится настолько разрежённой, что аэродинамическая авиация становится невозможной, так как скорость летательного аппарата, необходимая для создания достаточной подъёмной силы, становится больше первой космической скорости, и поэтому для достижения больших высот необходимо пользоваться средствами космонавтики
Ни о чем не сказало. Я же не говорю про высоту, может я про гиперзвук на 7.8км/сек в плотных слоях, лол. мысленный эксперимент, епта

>>328705
>Самолет по определению летает за счет подьемной силы крыльев. У тебя не самолет.
Ладно, защитано.
Аноним 02/03/17 Чтв 23:30:17  328707
>>328706
>Я же не говорю про высоту,
Суть той высоты, что сколько ты крыльев не вешай, на определенном моменте для того, чтобы быть самолетом (не падать за счет крыльев) у тебя должна быть орбитальная скорость. Т.е. на той высоте у тебя крылья подъемную силу считай и не дают.
А если ты ниже летишь на орбитальной скорости, то и подавно, они только довить будут чтобы ты из атмосферы не улетел (хотя как направишь, можешь апоапсис в луну поднять, лол)
>мысленный эксперимент, епта
Чому сразу мысленный-то? Врубаешь читы и вперед, кербачь на здоровье!
Аноним 03/03/17 Птн 00:38:50  328708
>>328274
Пшолвон! Такую годноту можно и в каждый тред постить.
Аноним 03/03/17 Птн 00:40:02  328709
Посоны, а что там с Юноной? Пизда накрылась мечтой, я так понимаю и подышала наладом?
Аноним 03/03/17 Птн 00:44:22  328710
>>328709
Ну не окончательно наладом.
>>325656
Аноним 03/03/17 Птн 09:51:12  328732
>>328687
В этом и дело, что их пруфы в принципе то не пруфы.
Притяжение они объясняют тем, что плоская земля направляется с неебической скоростью вверх тем самым держа всех на земле. Кто говорит совсем иное.

Смену времени суток двумя светилами. И их неебет, что если бы земля была плоская, то солнце освещало бы все независимо от того, где оно находилось.

Ну а остальное - вскукареки типа "ледяные стены охраняет мировое правительство, жидомассоны" и тд и тп.
мимо-тот-кто задал вопрос
Аноним 03/03/17 Птн 09:53:03  328733
>>328651
ТЫ ЧО, ЗВЕДЫ ЖИ НАРИСОВАНЫ И ИХ СПЕЦОМ ВРАЩАЮТ.

Как-то так.
Аноним 03/03/17 Птн 09:59:56  328735
>>328732
>Притяжение объясняют тем, что плоская земля направляется с неебической скоростью вверх
Но каким образом скорость, пусть даже неебическая, становится силой? Тут в таком случае должно быть ускорение, но тогда вопрос о пределе скорости. И вообще подразумевает ли плоская Земля сама по себе плоские Солнца, Марсы, Юпитеры, Луны? Или Земля одна такая за каким-то хером особенная? И существует ли в модели с несущимся в неведомые ебеня диском с ускорением 9.81 м/с2 гравитация. Блять, кто бы мог подумать, что натягивания плоскоземья на реальную физику может оказаться столь увлекательным занятием
Аноним 03/03/17 Птн 10:14:46  328738
Раз вы начали про плоские земли:
https://www.youtube.com/watch?v=U2bc7kZcpmQ
Аноним 03/03/17 Птн 10:41:24  328739
>>328734
Запросто. Нужно место с очень тонкой реальностью и бог которому надоели элипсы и прочее.
Ещё четыре слона и черепаха.
Аноним 03/03/17 Птн 12:56:00  328747
>>328573
Ты охуенен.
Аноним 03/03/17 Птн 16:22:17  328761
Вообще забавно, что, скажем Америка от нас далеко и ее не видно но туда люиди летают и там бывают. Хотя есть люди которые никогда Америку не видели. И вот забавно что в то место, которе большинство большинство россиян не видело, люди как нехуй делать летают. А на Луну, которую видно легко не летают (кроме десятка пиндосов)

То ет ьс на земле есть места, которых мы не видим, но бываем там, зато в жизни мы тысячи раз видели луну, но мы с вами (петарды) там небыли и не побываем
Аноним 03/03/17 Птн 18:22:18  328767
>>328761
>не побываем.

А хуй его знает, а?
Аноним 03/03/17 Птн 18:40:58  328770
Лол, действительно тупой тред, куда не посмотри - черные дыры, плоские земли, парадоксы близнецов и ответы под стать вопросам.
Аноним 03/03/17 Птн 19:12:57  328773
Безымянный.png (271Кб, 1068x539)
https://www.gazeta.ru/science/2017/02/17_a_10530449.shtml
Разъясните пожалуйста. Шумеры типа были отчасти правы ?
Аноним 03/03/17 Птн 19:52:21  328775
>>328773
Как ж вы заебали с шумерами!
То, что все сейчас называют считаю самоназванием некоей планеты общеизвестное имя Нибиру - нихуя не имя. Нибиру как отдельное слово обозначает пересечение. А по контексту текст о нибиру, что-то там про пересечение толи орбиты Юпитира какой-то звездой (в понимании наблюдателя с Земли), толи непосредственно пересечения с самой звездой Юпитера с возможным увеличением свечения из-за наложения света от двух светил. И всё.
Аноним 03/03/17 Птн 21:29:59  328781
>>328761
Да ты и Россию не видал и не повидаешь за жизнь. И помимо Луны ещё сотни небесных тел видать на небе, но никто не заявлял, что там был.
Non sequitur.
Учись в логику у этого господина >>328573
Снимаю шляпу, вряд ли смог бы сказать лучше.
Аноним 03/03/17 Птн 21:33:35  328783
14883736632950.gif (1641Кб, 1000x563)
Можно ли застрять в положение между гелио- и геостационарной орбитами, если поместить спутник висеть на поверхности SOI, как будто песчинка, между жерновами, которая крутиться под You spin me right now, baby, right now?
Аноним 03/03/17 Птн 21:35:55  328784
>>328783
Нельзя. Геостационар 36ккм, соя земли гораздо дальше луны (400ккм), а гелиостационара нет, где ты поверхность солнца видел?
Малаца, хороший тупой вопрос.
Аноним 03/03/17 Птн 21:52:58  328786
Kb-xGLXIlik.jpg (121Кб, 807x807)
>>328784
> а гелиостационара нет
В СМЫСЛЕ НЕТ?!
Земля вся находится в гелиостационарном положении относительно солнца. По моей задумке, геогелиостационар - это, например, точка конца SOI Земли, в которой спутник точь-в-точь вторит орбите Земли ну или системы Земля-Луна хвотиком, всегда находясь в положении неотстающего, но и никак не догонящего догоняющего. Но! Это только одной точке ориты Земли, а в противоположной ей наоборот - носиком. Ибо при этом параллельно и попутно соблюдается и такое правило, что относительно самой Земли, данный спутник всегда указывает на одну и ту же точку неба. Например, запущен спутник, названный "Бетельгейзе-xD". В день запуска к моменту достижения им точки покоя, с поверхности, которая как раз встала на одной линии наблюдения со звездой Бетельгейзе. И как бы Земля не крутилась, "Бетельгейзе-xD" всегда будет сливаться со звездой Бетельгейзе с поверхности Земли, при этом вращаясь на внешней границе SOI Земли, чтобы было возможно выполнение этого условия, собственно.
Аноним 03/03/17 Птн 21:57:09  328787
>>328784
Тащемта и у Марса нет моря, но есть уровень моря, например. И у Сатурна тоже, хотя у того и поверхности нет. Точно так же радиус и скорость вращения хорошо известны (с небольшой погрешностью, разные техники измерения дают разные результаты), и по крайней мере аналог стационарной орбиты для Солнца вполне возможен.

>>328783
Такая орбита у Солнца находится в 2 млн км от поверхности, весьма близко от него, так что нет. а вообще вопрос настолько тупой, что я его не понял
Аноним 03/03/17 Птн 21:58:15  328789
>>328786
>Земля вся находится в гелиостационарном положении относительно солнца.
Блялол. Нет.
Аноним 03/03/17 Птн 22:05:12  328792
prosto-muzhik-s[...].png (283Кб, 1636x1254)
EarthAtmosphere[...].jpg (69Кб, 685x599)
Анон, поясни за эту картинку. За счёт чего в вакууме и околовакууме берётся нагрев? Ведь чем выше, тем более разряженная атмосфера, тем меньше атомов, которые могут о тебя потереться и нагреть и тем меньше шансов нагрева. В чём прокол? Если вспомнить мужика, который с орбиты упал, с высоты, например, 400км, когда, на каких высотах (и из-за чего) у него будет наступать перегрев?
На других атмосферных планетах такой же М-образный график термоперепадов?
Аноним 03/03/17 Птн 22:11:22  328794
>>328784
>>328787
Насколько я помню у Солнца разные участки поверхности движутся с разной скоростью. Экватор быстрее, а у полюсов помедленнее или наоборот. Как-то так в общем.
Аноним 03/03/17 Птн 22:14:11  328795
>>328792
>В чём прокол?
В том, что в твоей логике Солнце погасло.
Ионизация атмосферного воздуха, посредством УФ от Слонышка, в основном. Плюс охуенный вклад магнитосферы. По сути дела это очень разряженная плазма.
Аноним 03/03/17 Птн 22:29:38  328797
>>328794
Так и у газовых гигантов то же самое. А скорость вращения, внезапно - известна.
Аноним 03/03/17 Птн 22:31:03  328798
>>328792
>За счёт чего в вакууме и околовакууме берётся нагрев?
Излучательный теплообмен с окружением - Солнцем, Землей, звездным фоном.
Аноним 03/03/17 Птн 22:33:11  328799
>>328792
>На других атмосферных планетах такой же М-образный график термоперепадов?
Зависит от конкретного тела, на разных в атмосфере происходят сильно разные процессы, так что не обязательно.
Аноним 03/03/17 Птн 22:45:01  328800
>>328799
А почему на Земле оно М-образное, а не допустим V-образное?
Аноним 04/03/17 Суб 06:27:59  328825
Очень тупой вопрос, https://www.youtube.com/watch?v=SF7FUU7CThs это они сейчас запись крутят? В десять вечера вчера астронавты ковырялись, и до сих пор продолжают?
Аноним 04/03/17 Суб 06:40:09  328826
>>328825
Не знаю что это за сомнительный канал такой, но официальный стрим прямиком с камер HDEV находится здесь: http://www.ustream.tv/channel/live-iss-stream
Более того, вообще-то МКС прямо сейчас находится в тени Земли, и с этих камер нихуя не видно.
Аноним 04/03/17 Суб 11:04:02  328832
>>328827
>1. Как попасть в другую вселенную?
Ты - набор состояний частиц тебя составлающих. В другой вселенной законы могут быть иными и твой набор будет нестабильным. Хелл, даже попытка телепортироваться в нашей вселенной может означать твою гибель по мнению некоторых философов. Да, философов, а что ты хотел? Физика не рассматривает непроверенные маняфантазии всерьёз.
>2. Как найти вселенную с более удобной физикой что бы не было проблем с энергией, скоростью света, зависимости от существования\не существования, проблемы с отсутствием ресурсов и прочей надоедающей фигни?
Мы даже не доказали существование других вселенных ещё.
>3. Как заставить органику так эпично с эволюционировать что бы она стала развиваться на равных с лучшими технологическими достижениями? Ну кароче, как Зергов создать? (Органика по прочности эквивалентна самым прочнейшим сплавам металлов, органические космические корабли и прочие мемасики)
Форс эволюции? Только ручками. А органика не сравнится со сталями-титанами, это ёбаное мясо и набор клеток, они не могут в кристаллическую решётку.
>4. Как развить пси потенциал цивилизации? Ну то есть суметь в телепатию, управлять материей силой разума, вознестись на высший уровень бытия и прочей фигнёй, или так и нет никаких подтверждённых данных на тему, что это вообще работает в нашей вселенной?
Хуита какая-то непонятная, невозможно ответить.
>5. Как выкачивать энергию из пространства времени нашей вселенной? Ну то пресловутое "энергия из вакуума", там же всякие виртуальные частицы рождаются и аннигилируют, эффект Казимира возникает, квантовое туннелирование из не откуда берёт энергию на свои шалости, квантовая запутанность, можно как то от этого запитать кудахтер\корабль\завод\чайник\вилку\генератор черваточин\варп двигатель?
Эффект Казимира нулевой по энергии, он не выделяет её. Последнее что я слышал про выкачивание энергий из вакуума были громкие вскукареки со стороны кефиротреда.
>6. Как создать стабильную нормальную червоточину диаметром в сотню метров, что бы она не разломала корабль приливными силами и прочим?
Никак, мы вообще не умеем в червоточины и они лишь предсказаны на основе метрик пространства-времени, у нас нет возможностей влиять на гравиполе, мы только недавно гравиволны улавливать научились.
>7. Как создать искусственный сверх интеллект, что бы он нас не вынес нафиг? Ну или хотя бы как то "мягко ассимилировал"?
3 закона робототехники. Но серьёзно - ты не можешь просто взять и написать ИИ, он должен основываться на постулатах и предпосылках для функционирования, как мы основываемся на инстинктах. Плюс должен быть механизм поощрения/наказания. Просто пропиши что люди хорошо и наказывай за битьё людей. Но лучше наоборот, пропиши что хорошо бухать и заставлять людей кусать твой блестящий металлический зад, это будет забавнее.
>8. Как сделать разумное существо в N раз умнее и могущественнее того, что оно может при своём генетическом и прочем развитии, то есть того чем оно в данный момент является?
Мы просто разумных существ особо не можем создавать, не то что модифицировать. На детей глянь.
>9. Как поглощать энергию\информацию\материю из других вселенных?
См. выше.
>10. Как контролировать всякую экзотическую материю\тахионную материю\тёмную энергию\материю и прочее?
Мы их не детектили ещё, это лишь матаппарат.
>11. Как суметь в существование во вселенных где измерений много больше трёх и черпать оттуда профиты?
Охуеваючи. Во вселенных с иными метриками набор состояний тебя будет вести себя иначе.
А вообще очень напоминает кефирные вскукареки. Если тебе нужны интересные маняфантазии - обратись к ним.
Откуда вообще нахватался подобного еленаучного говна?
Аноним 04/03/17 Суб 12:59:20  328835
Раз это тупых вопросов тред, то подкину свой:

Как заставить автора поста выше победить себя в старкрафте хотя бы раз в три тысячи попыток?
Аноним 04/03/17 Суб 13:08:01  328837
>>328833
>псионические способности в различной фантастике,
С антинаучной хуетой и прочей магией ты можешь проследовать в /mg/ и куда угодно, только сюда это говно тащить не надо.
>хитиновый покров не из кальция,
В хитиновом покрове нет кальция. Он блядь, как ни странно, из хитина. Это органическая молекула полисахоридов (C8H13NO5)n.
Раковины моллюсков крепкие потому что кальцит и арагонит в них не "вкраплениями", а в виде полноценной кристаллической решетки. Если металл будет вкраплениями - от него не будет никакого толку, он тоже должен быть выстроен в единую кристаллическую решетку. И тут мы идем в учебник химии и пытаемся найти белок, который может заменить конхиолин, который строит панцири молюсков и построить решетку из металлов, и не находим нихуя.

Аноним 04/03/17 Суб 13:11:24  328838
>>328833
>Статьи в интернете с самых разнообразных источников
Хватит жрать любое первое попавшееся говно без разбору. Содержи голову в чистоте. Ты кучу времени тратишь на всякую хуету, в итоге реальных знаний в голове нуль целых хуй десятых. Голова - это не помойка, не зачем носить туда мусор.
Аноним 04/03/17 Суб 13:40:45  328841
>>328833
>Ну... вообще то псионические способности очень часто показывают в различной фантастике
А ещё в медиа показывают цветных волшебных поняшек. Your point being?
>В общем суть вопроса: такие экзотические способности каких либо существ были когда либо обнаружены с нормальной фиксацией экспериментальных данных?
НЕТ. Отделяй реальность от фикшена, пожалуйста, потому, что это
>чистейшие маняфантазии
>Можно ли создать такие возможности у чего либо искусственным путём?
Зависит от. Я разрешаю. Физика - нет. Моделирование - да. Ты можешь надеть окулус, вставить всякие нанозонды и создавать/эмулировать миры. Но в нашем такого дерьма не замечено.

>Хм.. а возможны варианты симбиоза органики и не органики?
Невозможны по определению. Сим|биоз - совместная жизнь. Неорганика это не жизнь.

>Кстати неплохой пример, "относительно простое" существо превращается в куда более "умную и сильную" особь после взросления, можно ли как то ещё совершить настолько же качественный скачок развития, а потом ещё и ещё?
Возможно и стоило бы, но у нас моралфаги кукарекают, так что никакой тебе евгеники и cas/crispr на живых людях.

>Интересный маняфантазии я и сам придумать могу воображение у меня очень даже ничего так.
Здорово, я даже завидую слегонца. У меня есть воображение, но оно загнано в анальный огород рассудком и знаниями.

>Мне интересно, есть ли идеи у цивилизации как создать что то из того фантастического говна, что придумывают различные маняфантазёры.. Ну или может ли что то такое существовать без нашего творческого участия.
Постоянно всякое такое происходит. Планшеты в 60-х, транслятор в стар треке, ракеты Жюля Верна етц.

> Статьи в интернете с самых разнообразных источников
Боюсь, что эти статьи были как раз без источников.

>Я НЕ являюсь приверженцом кефирострадальцев, рептилойдофапов, эзотерико адептовъ и прочих и НЕ распространяю подобной хрени, но мне нравится различная "экзотика" и хочу понять стоит ли ожидать чего то не стандартного от этой вашей "реальности" во всех её аспектах.
Тогда ты охуенен. Читай/развивайся/спрашивай ответы и лелей свои мечты и в один прекрасный момент нагни физику как это сделали Исаак, Альберт, Стивен и сотни других известных ребят. Только что понял, что я знаю больше физиков, чем поп-звёзд. Я аутист?
Только прими к сведению - мы многое так и не узнаем за человеческую жизнь.
Поэтому ратуй за CAS/CRISPR и генную терапию когда тебе чинят старение и рак.
Аноним 04/03/17 Суб 14:12:15  328844
148769369215058[...].jpg (54Кб, 600x359)
Посматриваю тут сериальчик The Expance. По сюжету в одной из последних вышедших серий они преследуют астероид на военном корабле. Им нужно было ускориться до 20g и больше и капитан сказал "Вам пизда, ребятки", корабль на автопилоте, он будет лететь за астероидом, а мы можем сдохнуть. Почему так? Нельзя плавно ускоряться (на своей девяточке я могу вдавить педаль в пол (немного прижмет к сидушке) и разогнаться до 100км\ч, а могу плавно это сделать ничего не почувствовав)?
Аноним 04/03/17 Суб 14:24:19  328845
>>328844
Твоя девяточка выдаст тебе смешную десятую долю g даже при тапке в пол. Что-нибудь раллийное или тем более драговое заставит тебя уже малость охуеть с непривычки, и то там несколько секунд разгон, максимум. А 20g в течении сколько-нибудь продолжительного времени - это когда глаза вытекают через хуй. Такие перегрузки бывают например при срабатывании башни САС, но она работает 3-4 секунды может быть, и то травмоопасно пиздец, можно здоровья лишиться даже тренированному космонавту. Справедливости ради, на 20g можно разогнаться до ебических скоростей очень быстро, и долгий разгон не нужен.

>плавно ускоряться
Ускорение это и есть первая производная от скорости, т.е. ты плавно наращиваешь скорость. Вопрос только в том, насколько плавно.
Аноним 04/03/17 Суб 14:27:33  328846
>>328844
Нужда так ускоряться только спасаясь от взрыва на старте.
На космических дистанциях такой нужды тупо нет, ты можешь ускориться на 20г сейчас или денёк покрутиться с обертом на 0.05г люто сэкономив всё.
Аноним 04/03/17 Суб 14:36:59  328847
>>328846
У него там в сериале манясайфай и бюдющее без тирании циолковского, военные дроны маневрируют и летают по прямой когда хотят, вместо переходов и баллистических окон, так что такие ускорения там оправданы. потому что могут, ёба
Аноним 04/03/17 Суб 14:40:09  328848
господа, привлекает тема космоса, но в физике и астрономии я примерно 0 чтобы читать достаточно продвинутую литературу. что можно почитать чтобы было популярно, не сильно заморочено и при том интересно?
Аноним 04/03/17 Суб 14:41:04  328849
>>328848
Понятие крайне широкое. Сузь область - астрономия, космонавтика, что конкретно ты хочешь.
Аноним 04/03/17 Суб 14:42:17  328850
>>328849
астрономия скорее.
Аноним 04/03/17 Суб 14:46:22  328852
>>328850
Например Хокинг, Пенроуз, Грин (который Брайан). Годные популяризаторы физики и астрофизики, практически весь их научпоп читабелен и годен.
Аноним 04/03/17 Суб 14:46:25  328853
>>328851
тогда космологию. но мне кажется такое конкретное разделение будет в изданиях которые я не осилю в силу своей слабой осведомленности в теме.
>что можно почитать чтобы было популярно
Аноним 04/03/17 Суб 14:47:44  328854
>>328852
благодарю. погуглю, ознакомлюсь.
Аноним 04/03/17 Суб 14:49:35  328855
>>328848
Мир в ореховой скорлупке. Самый энтрилевел, все понятно и с картинками.
Аноним 04/03/17 Суб 14:55:31  328856
>>328855
и тебе тоже спасибо.
Аноним 04/03/17 Суб 15:25:38  328858
>>328833
У тебя примерно такая дискуссия тут

-Пацаны, а какого размера хуй у йети и у лохнесского чудовища?
-Их никто не видел, только маняфантации из-за невнятных фоток. Существование этих пиздюков не доказано.
-Но ведь йети очень часто показывают в различной фантастике! Удивлён что ты не сразу понял, ты че баран? Если вкратце то вот статья: ren-tv.ru/как-меня-выебал-снежный-человек-пока-я-наблюдал-за-лохнесским-чудовищем-но-меня-спасли-инопланетяне-с-альфы-центавра.html
Аноним 04/03/17 Суб 15:39:56  328862
>>328860
О, началось. Иди нахуй отсюда, шизик.
Аноним 04/03/17 Суб 15:44:23  328864
>>328860
На нашей планете доказательствами занимаются ученые. Сходи к ним и попроси научно обосновать твое существование. В чем проблема?
Аноним 04/03/17 Суб 20:53:35  328882
Поясните, если космонавт окажется в невесомости в центре комнаты так, что до стен он доставать не будет - он навеки застрянет в этом положении?
Аноним 04/03/17 Суб 20:55:23  328883
>>328882
Нет. Машешь руками, дуешь, добираешься до стенки. Если комната большая - неудобно, но не более.
Аноним 04/03/17 Суб 20:56:32  328885
>>328883
Т.е. просто медленно доберётся?
Действия руками дадут эффект?
А если он в скафандре - т.е. дуть не сможет?
Аноним 04/03/17 Суб 20:59:18  328887
>>328885
Руками тоже можно. Давай я доведу твой вопрос до логического завершения - что если эта комната наполнена сжиженным вакуумом?
Аноним 04/03/17 Суб 21:01:17  328888
Почему Ангара А5 тяжелее чем Falcon 9FT на 220 тонн? При сходной (и даже большей у Фалкона) выводимой нагрузке.

При этом двигатели Ангары примерно на 10-12% мощнее Фалкона. Это следствие запуска Ангары с Плесецка? Я пытался использовать формулу Циоловкского - но как раз и получилась ,что более мозные двигатели должны компенсировать разницу в широте Плесецка и мыса Канаверал. Итого - что такого тяжелого в Ангаре?
Аноним 04/03/17 Суб 21:34:24  328889
>>328888
Тяжелее значит надежнее, ты ничего не понимаешь. Это старая советская традиция, смотри Н-1 и S-V.
Аноним 04/03/17 Суб 21:37:26  328891
>>328885
>действия руками дадут эффект
Да. Хуевый, но дадут. Получится что-то вроде простейшего гиродина. Котейки всегда падают на лапы по схожей схеме - резко гнут или распрямляют спину в полете, можешь погуглить видево на трубе.
Аноним 04/03/17 Суб 21:38:22  328892
Есть какие-то продвижения по ведру?
Аноним 04/03/17 Суб 21:40:15  328893
>>328892
новостей давно не было
Аноним 04/03/17 Суб 21:45:04  328894
>>328893
Чисто кукаретически представим, что ведро подтвердилось и технология взлетела. С ее же помощью тогда вроде как можно вечный двигатель собрать? скотт мэнли рассказывал, но я забыл схему этой хуйни
Аноним 04/03/17 Суб 21:48:45  328895
>>328894
>С ее же помощью тогда вроде как можно вечный двигатель собрать?
Я не знаком с этой хуйнёй, но вибрации и устаревание материалов было сведут эту хуйню на нет. И вроде там выделения энергии просто мизер.
Аноним 04/03/17 Суб 21:49:03  328896
>>328894
Ты неверно понимаешь суть вечного двигателя.
Аноним 04/03/17 Суб 21:49:13  328897
>>328894
Примерно такой же вечный, как и солнечные батареи. Мясным мешкам хватит за глаза, в общем.
Аноним 04/03/17 Суб 21:52:44  328900
>>328896
Верно, ведь оно якобы тянет больше, чем давление излучения.
Аноним 04/03/17 Суб 21:58:53  328904
>>328888
Массовое совершенство, + экстремальное переохлаждение (эти наркоманы даже керосин холодят, а кислород закончился-таки бахом)
Аноним 04/03/17 Суб 22:00:58  328906
>>328904
Разверну - 5 модулей и 1, 2.5 ступени против 2, выводы делай сам. Алсо, центральный троттлит больше, отсюда потери импульса.
Аноним 04/03/17 Суб 22:05:10  328907
>>328892
>>328893
>>328894
В ведротред съебите, пидорасы, и ебите друг-дружку там.
Аноним 04/03/17 Суб 22:07:00  328908
>>328906
>>328904
Я прошу прощения, может совсем тупой, но ничего из постов не понял. Кто кого холодит? Какие 5 модулей? Они же обе на жидком кислороде и керосине. И главное, масса-то откуда?
Аноним 04/03/17 Суб 22:31:42  328911
>>328908
Чем меньше массовое совершенство, тем больше нужна общая масса ракеты (ракета+топливо) для подъема одной и той же ПН. У фалкона-9 первая ступень это один модуль, у Ангары А5 - 5 штук, соответственно паразитной массы больше. Кроме того, в А5 центральный модуль продолжает работать после отделения боковушек, а топлива там одинаково. Т.е. в начале полета его дросселируют (ограничивают тягу), что всегда означает потерю удельного импульса из-за неполного сгорания. И у фалкона FT кислород переохлаждают до упора, чуть ли не до точки затвердевания, от этого он становится более плотным, а ракета остается той же, = массовое совершенство еще выше. Даже керосин немного холодят, хотя толку от этого минимум, он плохо ужимается. Вот и выходит у А5 лишняя масса, даже несмотря на то, что Мерлины имеют удельный импульс ниже, из-за своего открытого цикла.
Аноним 04/03/17 Суб 22:41:45  328914
>>328911
Два литра лидского кваса тебе!
Аноним 04/03/17 Суб 23:50:44  328927
vve8mhSKDd0.jpg (28Кб, 607x335)
Только что навернул вот это: https://www.youtube.com/watch?v=MlIxlwWM3zs
Ближе к концу мужик-лектор упомянул, что в начале 90х открыли такую вещь, мол, раньше, до 90х, вселенная замедляла своё расширение, а сейчас вдруг наоборот попёрла с ускорением. Чому так? Как это вообще возможно?
Аноним 05/03/17 Вск 00:28:16  328929
>>328927
>раньше, до 90х
Кек. Ну хули, застой же был, а потом приватизация, капитализм, все дела.

Не до 90х. И не замедляла, а просто расширялась не так быстро. Ну короч тёмная энегрия. Понятней стало? Нет? Ну вот учёным не легче.
Аноним 05/03/17 Вск 00:28:47  328930
>>328927
Померили скорость, выяснили что расширяется с ускорением. Получили за это Нобелевку. Теперь думают от чего и почему. Условно назвали темной энергией.
Аноним 05/03/17 Вск 00:51:23  328935
142840795119074[...].JPG (186Кб, 1600x1200)
>>328930
Спорят как-то физик и экономист:
- Один плюс один равно три, - заявляет экономист.
- Что? Ты что, дурак? У тебя не может быть три, один плюс один образуют два! Чтобы у тебя было три, надо добваить ещё четыре по ноль двадцатьпять, или два по ноль пять или всего лишь один раз по одному
- Нет никакой проблемы! Один плюс один равно три.
- Да как ты вообще это объяснишь? Ты должен объяснить, откуда у тебя ещё одна единица или у тебя будет два, а не три, то есть неравенство.
- А мне ничего объяснять и не надо. Я просто буду действовать как вы - физики, скажу, что я что оно работает, просто я не знаю как.
- Что?!
- Да, это просто тёмное число, вам не понять, но оно реально работает и всё объясняет.
Аноним 05/03/17 Вск 00:59:09  328937
EngleB20130405l[...].jpg (85Кб, 600x421)
>>328935
Аноним 05/03/17 Вск 01:59:41  328941
Нужно клонировать Эйнштейна чтобы все разрулил.
Аноним 05/03/17 Вск 02:52:17  328943
>>328937
Влияние темной материи заметно благодаря гравитационным линзам, а вот с темной энергией все пока совсем-совсем плохо.
Аноним 05/03/17 Вск 03:08:55  328946
>>328943
> Влияние божественного начала заметно благодаря чудесам, а вот с духом святым все пока совсем-совсем плохо.
Аноним 05/03/17 Вск 03:27:28  328947
>>328946
Сейчас ОТО-блядская моча придет и зобанит за расжигание
Аноним 05/03/17 Вск 03:34:02  328949
>>328947
Но ведь серьёзно, это же блядский цирк. Так бы и сказали, наши теории были неверны, мы не знаем как устроена вселенная, но будем над этим работать. Это все равно что я на работе вместо того чтобы искать поломку буду молиться богу машины.
Аноним 05/03/17 Вск 04:08:48  328950
>>328825
Да, судя по всему говнозапись, сам сначала подумал "ого, прям щас в открытом космосе шарятся, охуенно", ан нихуя.
Аноним 05/03/17 Вск 06:29:10  328951
>>328949
Ты немного неправильно понял, темная материя это наблюдаемый факт. Есть кончено модели которые позволяют без нее обойтись, но они совсем унылые и притянутые за уши. Лет через пять-десять, когда будут научные данные с Уэбба и еще кучки других инструментов вроде гамма-детекторов, с темной материей, скорее всего, разберутся. Либо найдут ее саму, либо хотя бы поймут где искать, коллайдер там новый построить или еще какую штуку.
К темной энергии, однако, не знают даже как подступиться. Это Неведомая Ебаная Хуйня как она есть.
Аноним 05/03/17 Вск 11:38:32  328955
79f8325df92d241[...].gif (2511Кб, 500x497)
>>328951
> Ты немного неправильно понял, темная материя это наблюдаемый факт.
У тебя немного подмена понятий. Наблюдаемый факт - вещество в известной нам вселенной ведёт себя не так, как оно должно себя вести согла нашим рассчётам. Всё. А вот то, что значит там есть что-то, чего мы не видим, но что влияет, например, некая неуловимая ни одним датчиком, например некая тёмная материя, это называется домыслы и спекуляции.
Аноним 05/03/17 Вск 12:20:51  328958
>>328955
> У тебя немного подмена понятий. Наблюдаемый факт - металл в известных нам розетках ведёт себя не так, как он должен себя вести согла нашим рассчётам. Всё. А вот то, что значит там есть что-то, чего мы не видим, но что влияет, например, может ёбнуть если взяться за два провода, например некое электричество, это называется домыслы и спекуляции.
Аноним 05/03/17 Вск 12:22:05  328959
>>328958
Теория нинужна. Ясно.
Аноним 05/03/17 Вск 12:24:42  328960
>>328959
Не знаю что тебе там ясно у тебя в голове, но ты о чём-то своём.
Аноним 05/03/17 Вск 13:44:11  328968
>>328955
Это не домыслы и спекуляции, это НАУКА, она так и работает. Есть неведомая ебаная хуйня - придумаем десяток теорий почему она такая и отсеем их, самую жизнеспособную назовем действующей теорией и она будет таковой пока не опровергнута
Аноним 05/03/17 Вск 13:51:11  328969
Долго еще тут будете кефироблядей раскармливать, поехавшие?
Аноним 05/03/17 Вск 13:52:28  328970
>>328969
Они напрямую про кефир не говорят.
А если это не кефир, то почему бы и нет?
Аноним 05/03/17 Вск 13:57:39  328972
14807731915821.webm (1776Кб, 240x180, 00:01:14)
astrophysics.png (26Кб, 579x314)
>>328955
Это жизнеспособная теория. Подтверждаемая расчётами. Да, в ней есть противоречия, но на данный момент её опровергнуть сложнее, чем любую другую альтернативную теорию.
Ты называешь это спекуляцией потому, что это не укладывается в твою привычную систему мира. Но Вселенной поебать на сознание каких-то там эволюционирующих обезьян. Она просто продолжает работать и не стремится упростить свою природу ради нас.
Гугли "скопление Пуля", если хочешь наблюдаемые факты.
Аноним 05/03/17 Вск 14:00:46  328973
>>328972
>Ты называешь это спекуляцией потому, что это не укладывается в твою привычную систему мира
Это. Достаточно представить микромир, никаких шариков на орбите других шариков, никакого света, гравитацией можно пренебречь етц. Неподготовленный мозг лопается. Так и в макромасштабах тоже непривычная мясным мешкам поебень. Понятно, что будут бугуртить и кричать ВРЁТИ и выдумывать даже плоские земли лишь бы не было сложно
Аноним 05/03/17 Вск 17:48:52  328986
Какого хрена всё вращается? Галактики, звёзды, планеты. Не вокруг друг друга, а по своей оси какой-нибудь. Вот было облако газа, стало оно звездой и она тут же вращается. Без вращения не живётся что ли?
Аноним 05/03/17 Вск 17:50:16  328987
>>328986
СУЕТЛИВАЯ ГОЛАКТЕКА
Аноним 05/03/17 Вск 18:46:03  328995
пик.png (30Кб, 950x874)
Реквестирую такой пик эволюции звёзд. Набросал в пэинте что запомнил.
Аноним 05/03/17 Вск 19:26:24  328997
>>328065 (OP)
Аноны, в каком-то из здешних тредов привели ссыль https://ru.wikipedia.org/wiki/%D0%92%D1%80%D0%B5%D0%BC%D0%B5%D0%BD%D0%BD%D0%B0%D1%8F_%D1%88%D0%BA%D0%B0%D0%BB%D0%B0_%D0%B4%D0%B0%D0%BB%D1%91%D0%BA%D0%BE%D0%B3%D0%BE_%D0%B1%D1%83%D0%B4%D1%83%D1%89%D0%B5%D0%B3%D0%BE. Прошел и прочитал ахуев от собственной ничтожности, конечно.
Вопрос такой: к чему сейчас больше все же физика склоняется - протон все же распадается или не, какого из двух сценариев ожидать?
Аноним 05/03/17 Вск 19:33:25  328998
530094339563352[...].jpg (56Кб, 667x858)
facepalmbear2-t2.jpg (22Кб, 270x203)
images.jpg (4Кб, 318x159)
yetanotherpicar[...].jpg (27Кб, 448x352)
>>328995
Аноним 05/03/17 Вск 19:35:22  329000
>>328997
Ни к чему не склоняется. Нет достаточного количества данных, что бы ответить на вопрос.
Аноним 05/03/17 Вск 19:35:55  329001
>>328986
Нет, не живется. Все вращается.
Аноним 05/03/17 Вск 19:38:04  329002
>>329000
Ясно. Знач как всегда - есть два стула...
Аноним 05/03/17 Вск 19:39:41  329003
767124.jpg (105Кб, 753x612)
>>328995
Самая полная, что я видел. Без учета экзотических вариантов.
Аноним 05/03/17 Вск 19:46:49  329006
>>328997
> протон все же распадается или не
Ни разу не распадался, не наблюдали. Может у него гугол лет на это требуется, а может и вовсе не распадётся.
Аноним 05/03/17 Вск 19:52:28  329008
>>329001
почему?
Аноним 05/03/17 Вск 19:54:36  329009
>>329008
Потому, что ПОШЁЛ НАХУЙ, ВОТ ПОЧЕМУ так получилось. Если бы вселенная была равномерненькая без флуктуаций, то хрена с два вообще галактики со звездами собрались бы. Но тут и там были мелкие подвижки в ту или иную сторону и вот это движение никуда не денется, закон сохранения, сучечка.
Аноним 05/03/17 Вск 20:04:49  329010
>>329009
Т.е. можно так сказать, что вращение объектов это аксиома вселенной? Оно ничем не объяснено просто есть и всё?
Аноним 05/03/17 Вск 20:12:13  329012
>>329010
Оно появилось т.к. вселенная неоднородна. Почему она неоднородна - вопрос другой. Но да, смирись с этим так же, как смирись со скоростью света и гравитационной постоянной.
Аноним 05/03/17 Вск 21:49:40  329018
>>329003
протозвёзды бы ещё мастера фотошопов прикрутили
Аноним 05/03/17 Вск 22:26:06  329021
1 (2).jpg (200Кб, 1536x1208)
Может ли атмосфера планеты целиком состоять из воды?
Аноним 05/03/17 Вск 22:37:56  329022
>>329003
> Brown dwarf -> Brown dwarf

Чёт кекнул.
Аноним 05/03/17 Вск 22:49:46  329023
Вселенная - это обьемная черная пустота, в которой находятся галактики. То есть, я от Млечного Пути полечу куда-нибудь, вне зависимости от направления, то все равно наткнусь на че-нибудь?
Аноним 05/03/17 Вск 22:52:00  329024
>>329021
Нет, не может. Потому как даже если какой-то особой магией удалить все остальные газы, то космическое излучение будет пидорасить молекулы воды на водород и кислород.
Кстати спейсач - вот вопрос Аноним 05/03/17 Вск 23:38:57  329025
>>329003

Чтобы сформировалась планетарная туманность - рядом должно было неслабо ебнуть. Причем рядом.

Впорос - а где тело? То есть, а где труп? По идее должен быть труп звезды или звезд, которые дали жизнь SOL. Почему мы его не нашли? И вообще, куда деваются трупы? Почему мы вон уже изучаем зеленых буказоидов на Траписте, чекаем звезды а другом конце галактики, а вот трупов так и не нашли???
Аноним 05/03/17 Вск 23:39:02  329026
>>329024
А полностью покрытая жидкой водой?
Аноним 05/03/17 Вск 23:53:19  329027
>>329026
Как нехуй, гугли "планета-океан".
Аноним 05/03/17 Вск 23:55:11  329028
>>329026
Нет, потому что жидкая вода лишившись давления начнет выкипать при уже очень невысокой температуре.
>>329025
>планетарная туманность
В планетарных туманностях звездные трупы и находят, что бы ты знал.
Просто ты путаешь планетарные туманности и туманности со звездообразованием, то есть зоны H II, а это нихуя не одно и тоже.
Аноним 05/03/17 Вск 23:57:31  329029
>>329028
>>329026
В смысле покрыта водой без газовой оболочки не может. А так да, планета-океан.
Аноним 06/03/17 Пнд 00:03:28  329030
>>329028

Эм, ну чтобы сформировалось солнце - необходим был бабах кого-то большого и голубого. Причем этот бабах должен быть недалеко от нас - уровня ближе АльфыЦентавра.

Ну и где тело этого большого и голубого?
Аноним 06/03/17 Пнд 00:14:03  329031
CaXtoYSW4AA4zG6.jpg (75Кб, 604x591)
>>329030
Эмм, вот непонятно: ты ведь не шаришь нихуя, но при этом кидаешься какими-то утверждениями довольно уверенно.

Звезды образовываются в молекулярных облаках, которые в основном состоят из водорода и никаким боком к планетарным туманностям не относятся. Молекулярные облака давным давно обогащены металлами и постоянно обогащаются, если, правда, близкиt взрывs сверхновых не сдувает их к хуям.
По поводу того, почему рядом с нами не летает какой-нибудь огарок, то видишь ли в чем дело: Солнце давно уже съебалось с того места, где было рождено. Насколько далеко - ответ открытый. GAIA пытается найти сестринские звезды, но пока выводы делать рано.
Аноним 06/03/17 Пнд 00:25:33  329032
>>328795
>>328798
>>328799
Так, а если я хочу мужиком, пускай и в скафандре, упасть с НОО и приземлиться на поверхность Земли живым и максимально невредимым, я не смогу из-за этих перепадов давления/температуры?
Аноним 06/03/17 Пнд 00:31:30  329033
>>329032
Не сможешь, потому что сгоришь нахуй. Но не в верхних слоях, потому как несмотря на температуру плазмы, она там довольно разряжена, а потому что нагреешься в плотных. Как ты собрался сходить с орбиты? Чем тормозить собрался с 8 км\с? Предячим паром?
Аноним 06/03/17 Пнд 00:37:50  329035
PentDvPQWZs.jpg (230Кб, 958x960)
>>329033
> Как ты собрался сходить с орбиты? Чем тормозить собрался с 8 км\с? Предячим паром?
Не знаю. Да хоть на снаряде базуки сяду, как Гарри Поттер на метлу и буду тормозить, на сколько получится. Мне ведь по сути нужно будет только в идеале тормознуть до нуля, чтобы падать перпендикулярно поверхности, то есть с "положения полудня", а по факту достаточно просто нырнуть под атмосферу, чтобы неминуемо затормозить. Но речь не о том. Допустим, я уже затормозил. В одном случае, идеально ровно, в другом - с аэротормозом. Плотные слои ведь будут у самой поверхности, по-идее. А там, наверху, небось и тормознуть как-нибудь чем-нибудь своё падение в приницпе можно будет.
Может быть, несколько парашютом мне помогут, если их использовать по очереди.
Аноним 06/03/17 Пнд 00:44:42  329038
0613181.jpg (33Кб, 500x260)
>>329035
>а по факту достаточно просто нырнуть под атмосферу, чтобы неминуемо затормозить.
>Плотные слои ведь будут у самой поверхности, по-идее.
А посоны с Колумбии-то и не знали.
Аноним 06/03/17 Пнд 00:58:50  329040
Пейсач, почему гугль-мапс не отрисовывает Московию в 3д? В США даже самые крохотные мухосрани доступны в 3д. Да ладно бы США, там понятно, но даже всякая срань на юге Португалии доступна в 3д. А Московия еще и в разрешении уебищном. Такому есть объяснение политико-технико-экономическими причинами? Не санкции же. Неужели неоправданно?
Аноним 06/03/17 Пнд 01:00:16  329042
>>329038
Может они просто слишком глубоко нырнули с куда большей высоты, чем НОО, поэтому и скорость при приближении к поверхности у них была значительно выше, и от этого им было значительно жарче.
И вообще, я ж видел, есть какая-то жаростойкая макроподушка для человека, похожая на растянутый воланчик.
Аноним 06/03/17 Пнд 01:02:10  329043
>>329042
Ты так и не ответил, чем ты тормозить собрался.
Аноним 06/03/17 Пнд 01:04:50  329044
>>329040
Почему ты спрашиваешь об этом здесь, а не в офисе гугла?
Аноним 06/03/17 Пнд 01:07:20  329047
source.gif (2335Кб, 570x244)
>>329043
Выпустил парашют один - стропы натянуло, скорость упала, парашют оторвало - выбросил б/у парашют, стал легче, падаешь медленнее - выпустил парашют два - стропы натянуло...
Аноним 06/03/17 Пнд 01:14:40  329049
>>329047
Выпустить парашют на 8 км\сек? Ты серьезно?

>выбросил б/у парашют, стал легче, падаешь медленнее
Лолблядь, на физике в школе че делал? Водку пил?
Аноним 06/03/17 Пнд 01:17:59  329050
>>329049
Получал трояки.
Это что, тред "Выпытай у анона как он учился в школе" или тупых вопросов тред?
Тем более - ещё раз - если я полностью сведу к 0 орбитальную скорость, я буду падать строго вниз.
Аноним 06/03/17 Пнд 01:26:02  329052
>>329050
>если я полностью сведу к 0 орбитальную скорость
Вот я и спрашиваю, как ты собрался сводить ее к нулю? Просто интересно.

Ладно, учитывая, что НОО - это примерно 200 км, то падая с такой высоты, не встречая заметного сопротивления воздуха ты довольно быстро преодолеешь звуковой барьер, (Баумгартнер, например, преодолел его за 50 секунд, а прыгал с гораздо меньшей высоты) а дальше тебе станет очень неприятно, когда ты таки наберешь пару-тройку махов и сваришься к хуям, или отбросишь кони от перегрузок.
Аноним 06/03/17 Пнд 01:30:25  329053
>>329052
Достаточно гасить по 40 м/с за километр полёта.
Аноним 06/03/17 Пнд 01:36:09  329054
>>329052
> Вот я и спрашиваю, как ты собрался сводить ее к нулю? Просто интересно.
Оседлал ракету или сцепленные огнетушители.
> (Баумгартнер, например, преодолел его за 50 секунд, а прыгал с гораздо меньшей высоты)
Ок, а как он с этим справился?
Аноним 06/03/17 Пнд 01:42:54  329055
>>329054
Справился с чем?
Аноним 06/03/17 Пнд 01:44:00  329056
>>329055
С тем чтобы не надавать тебе за щеку
Аноним 06/03/17 Пнд 01:46:15  329057
>>329056
Идиот.
Аноним 06/03/17 Пнд 01:49:22  329059
>>329057
Ну мы же все таки на двачах, пусть и на острие науки.

Да и вообще, как что-то плохое.
Аноним 06/03/17 Пнд 01:52:42  329060
0oMhDdnZScA.jpg (37Кб, 684x386)
>>329055
Ты троллишь или даун?
> (Баумгартнер, например, преодолел его за 50 секунд, а прыгал с гораздо меньшей высоты)
> контекст о ебических перегрузках
С чем же он, блядь, справиться-то мог?! Ну я даже не знаю! С попыткой приготовить творожники без творога, чем же ещё ему заниматься в верхних слоях атмосферы-то?
Аноним 06/03/17 Пнд 01:53:42  329061
>>329060
Так он здоровый мужик небось был.
Аноним 06/03/17 Пнд 01:55:26  329062
Как ты, блядь, [...].webm (1038Кб, 640x360, 00:00:07)
>>329061
Аноним 06/03/17 Пнд 02:10:46  329064
>>329040
Потому что там их нарисовали, а тут не нарисовали.
вроде была еще какая-то машинная хуйня, но она плохо работает
Аноним 06/03/17 Пнд 02:18:27  329065
>>329054
Во-первых с трудом, во-вторых все-таки 1300 км\ч еще как-то жить можно. Больше чем 1340 он так и не набрал. Учитывая, что он был крепкий парень и все такое.
>>329053
Ты еще один идиот? Ты понимаешь, что Колумбия к хуям развалилась уже на 80 км при 5.2 км\сек? К тому времени шаттл уже пиздец как разогрелся.
Аноним 06/03/17 Пнд 02:21:00  329066
>>329032
Вообще не понял какая связь между орбитой, приземлением и температурой.

1. В вакууме в скафандре ты быстро спечешься даже в тени - тело генерирует собственное тепло быстрее чем отдает вовне. (если у скафандра не предусмотрены какие-нибудь раскладные радиаторы)

2. Если ты попытаешься сойти с орбиты в скафандре любым способом - погасив несколько десятков м/с или все 7.8км/с, ты сгоришь в атмосфере, но за счет быстрого движения в плотных слоях, а не потому что тебя будут эти горячие ионы жарить. Эти температуры тут ни при чем, там где жарко - воздух слишком разреженный, чтобы это как-то ощущалось.
Аноним 06/03/17 Пнд 02:22:54  329067
>>329065
Если гасить скорость как я писал то на 80 км будет всего лишь около 3 км/с.
Аноним 06/03/17 Пнд 02:24:19  329068
>>329067
Ты к тому времени уже дохлый будешь.
>всего лишь около 3 км/с
>всего лишь
Вообще охуеть.
Аноним 06/03/17 Пнд 03:39:09  329072
У меня есть вопрос: почему SpaseX использует в своих ракетах криогенное топливо ебучаяя транслитерация а не высококипящие? Ебли с ним меньше, плотность выше, движки проще.
Аноним 06/03/17 Пнд 03:49:58  329073
>>329072
Ификтивние.
Аноним 06/03/17 Пнд 03:53:55  329074
>>329072
Ну и вопрос. Никто не будет в 21 веке в здравом рассудке без специфической нужды (вроде МБР на боевом дежурстве) ебаться с практически БОВ. Не уверен даже, что им бы разрешили регуляторы. К тому же импульс говно, а для плотности/тяги петарды есть. Никто давно в качестве топлива для новых РН его не рассматривает, тем более в реюзабельном аппарате. Даже китайцы выводят из обращения.

>Ебли с ним меньше
Ага, щас.
Аноним 06/03/17 Пнд 03:55:37  329075
9893178.jpg (40Кб, 640x304)
>>329072
Ну Валентин Петрович...
Аноним 06/03/17 Пнд 04:02:29  329076
>>329071
>Хм интересно а если добавить размера и массы? Ну там диаметром с двое земных или больше и массы чтоб гравитация бала 5 или 10G
Похуй вообще. На поверхности-то давления не будет = будет испаряться. А пар просто сформирует атмосферу, и будет кислород и водород, как сказали выше, причем водород будет улетучиваться.

>50 или там 100G
Такая ёба уже зажжется звездой. (а если не зажглась бы, все равно похуй)
Аноним 06/03/17 Пнд 04:04:06  329077
VP.jpg (31Кб, 170x255)
>>329075
Ну пацаны, харэ меня тралить, я к середине семидесятых за кислород-водород топить начал.
Аноним 06/03/17 Пнд 04:27:39  329078
14089708924110.gif (1927Кб, 380x214)
>>329060
>контекст о ебических перегрузках
Каких блять перегрузках, че ты несешь? Он падал в постоянном поле тяготения планеты. Ты вообще понимаешь что такое перегрузка? В постоянном поле, даже несмотря на наличие ускорения, перегрузок нет, потому что на каждый атом тела поле воздействует с одинаковой силой, что в отсутствии опоры фактически ощущается телом как невесомость. Единственные перегрузки, которые он испытал, так это торможение со сверхзвука при погружении в более плотную атмосферу. Но это блять за сотню-две секунд он тормозил с 330-350 м/с, до стандартных парашютных 50-60. Это блять усредненные 2-3 м/с2, или 0.3g, то есть треть от стандартного земного притяжения, которое является для человеческого организма нормой, если не падать вниз ногами.
Аноним 06/03/17 Пнд 07:34:24  329081
>>329023
Смотря как полетишь. Если фотоном, то можешь так и летать пока вселенную не распидорасит.
Если мясным мешком в кастрюле, то быстрее сыграешь в ящик. Если сферическим бессмертным мясным мешком с помощью реалистичных технологий, то с большей вероятностью засосёт в какую-нибудь мимогалактику.

А вообще - не рекомендую летать в межгалактическом пространстве на досветовых скоростях, это скучно. Проверь в Спейс Энжине.
Аноним 06/03/17 Пнд 07:40:21  329082
>>329069
Потому, что это просто сверхновая, наверное, и ничем, кроме с потолка взятых размеров от нее не отличается?
Аноним 06/03/17 Пнд 07:45:15  329083
>>329072
Пошел нахуй, гептилоблядок хуев. Белые люди давно на керосин перешли, а скоро и на метан подсядут.
Аноним 06/03/17 Пнд 08:39:01  329086
>>329081
>имблаинг что летать на световой скорости намного веселее
Аноним 06/03/17 Пнд 10:49:56  329094
В теории можно изменить геном человека так, чтобы ему было плевать на радиацию?
Аноним 06/03/17 Пнд 11:46:01  329095
1 (2).jpg (26Кб, 590x251)
>>329094
Можно ли изменить человека так, чтобы он в расплавленном металле не дох?
нельзя, достаточно сильное ионизирующее излучение пидорасит любые сложные молекулы, ему похуй что твоя мамаша - шлюха
Аноним 06/03/17 Пнд 11:53:24  329096
234412144223433[...].jpg (54Кб, 597x336)
>>329094
Ну тут сложно. Вопрос в границе человечности. Когда плод генной инженерии еще человек, а когда уже проект #307 "антимутагенный". А вообще конкретики мало. Что значит "плевать на радиацию?" Это равносильно что сказать "плевать на температуру". Какую температуру-то? 10 по Кельвину или 1030 по Кельвину? Нужно же рамки какие-то подразумевать. А вообще с определенных температур перестают существовать молекулярные связи, с других определенных уже атомные связи. Само собой человеку достаточно небольших по вселенским масштабам температур, чтобы молекулярные связи огромных органических молекул в клетках рвались и переставали выполнять свои жизненно необходимые функции.

Радиация в каком-то смысле тоже самое, что и температура. Перенос энергии частицами или волнами. Частицы или волны либо просто разрушают молекулы, либо ионизируют атомы в молекулах клеток. Ионизированные атомы начинают шалить в нейтральных по заряду в целом молекулах, разрушая их или создавая свободные радикалы, те самые, с которыми призваны бороться разрекламированные антиоксиданты. По ходу дела свободные радикалы также могут создавать непредвиденные соединения в цепочках ДНК. Или само излучение может нарушать структуры ДНК определенных участков клеток. Клетки начинают делиться по механизму "сломанной" ДНК. Вроде как, насколько я пониманию, многие такие мутировавшие клетки отмирают как нежизнеспособные, потому что изменения в ДНК привели к критической ошибке. Или уничтожаются иммунитетом организма как вредоносные. Но бывают и мутации с печальными последствиями, когда ошибка в ДНК не достаточно критична в ближайшей перспективе и клетки начинают делиться с приоритетом полноценных клеток организма и иммунитет не воспринимает их как врагов.

Однако клетки повреждены и нихуя не выполняют своих жизненно необходимых функций. Пресловутые опухоли. Доброкачественные подразумевают собой некие структуры клеток, которые не совсем ушли в разнос и частично сохранили свои функции. Злокачественные - те, которые совсем поломались нахуй и творят неведомый пиздец, имея при этом приоритет на долю ресурсов организма как у нормальных клеток. Однако пускающие этот ресурс не на работу во благо организма, а на прямое вредительство, превращаясь в ткань-паразита.

Это что касается длительных последствий ионизации. Не биолог, не знаю можно ли выискать некий ген резистентности к патологическим процессам, чтобы как-то культивировать его и получить профит в целом. Но сомневаюсь. Ведь когда ломается структура ДНК, тогда может сломаться и сам этот ген, превратившись в непонятное нечто. Может есть некий ген, ответственный за "разумность" иммунитета, который позволит лейкоцитам эффективней бороться с подобными процессами. Но скорее изящным решением было бы создание некоего искусственного иммунитета из миллиардов миллиардов нанороботов, которые лазили бы по всем молекулам ДНК в теле и проверяли бы его на соответствие изначальному шаблону. И в случае несоответствия чинили бы поломки с помощью припасенных атомов. Это кстати, возможно позволило бы биологической оболочке и вовсе не стареть, хотя в механизмах старения я не разбираюсь ну совсем. Но если кто-то взломает нахуй систему и поменяет паттерн изначальной ДНК на ДНК например собаки, то человек умрет нахуй от жуткой опухоли в форме зародыша собаки или в форме кожи, превратившейся в шкуру собаки, типа того, хуй знает. Крипота пиздец, лучше сознание в сеть переселить, мешки с костями не нужны.

Что касается больших доз излучения в краткосрочной перспективе, то тут уже мало чем можно помочь. Если доза реально большая, то никакой сверхразумный наноиммунитет не поможет. Даже если нанороботы быстро будут восстанавливать уничтоженные клетки, то мозг наверняка выдаст критическую ошибку и организует самоуничтожение.

А если серьезно, то при большой единовременной дозе очень много тканей будет просто уничтожено на клеточном уровне. Молекулы сломаются и не смогут выполнят функции химических трансмиттеров, кислород не будет поступать в кровь, не будет поступать в мозг, мозг не будет посылать команды мышцам и сердцу в частности. Фатал еррор. Если доза большая, но не единовременно смертельная, то тканей будет также много повреждено, но частично функционировать организм будет. Однако большое количество клеток также мутируют через промежуток времени и наверняка перестанут функционировать, что уже окончательно приведет к смерти. Если же не приведет, то в не самой далекой перспективе онкология наверняка настигнет. Через пять лет после бомбардировок Хиросимы и Нагасаки число погибших от рака в результате облучения превысило число погибших от взрывов.

Сложно сказать как тут может помочь изменение генома. Возможно ген, ответственный за регенерацию тканей мог бы помочь справиться с последствиями "ожога". Но в целом сам основной вред излучения кроется либо в фатальной критической дозе, откатить последствия которой биологически просто невозможно ввиду специфики процессов. Либо в долгосрочной перспективе. И тогда это звучит чем-то вроде "можно ли вмешаться в геном так, чтобы любые другие вмешательства стали невозможны". Вероятно нет. Но опять же я далеко не специалист и совсем не разбираюсь в механизмах работы стволовых клеток и в различиях между ДНК в ткани кишечника и ДНК в носовых волосах. Может зайдет специалист-теоретик какой, получше пояснит.
Аноним 06/03/17 Пнд 11:59:10  329097
>>329078
Перегрузки там достигали 3.5G, в основном когда его закрутило, а не когда тормозило, он чуть сознание не потерял. (ну и при вытягивании парашюта, но там секунд 5 от силы) Это тебе не в противоперегрузочном кресле кулем сидеть.
мимо

Закрутка вообще главный враг парашютиста. Но на гиперзвуке если входить, там есть эффект самостабилизации вращения после 6.5 кажется махов- так что сгоришь не вращаясь.
Аноним 06/03/17 Пнд 12:50:25  329099
3b57cba0783d.png (42Кб, 300x293)
>>329097
Да, посмотрел вот, чет как-то прежде особого интереса к прыжку не испытывал, не в теме был. Вообще 3,5жэ не очень много, но крутило его действительно нихуево. А при карусельке в плоскости выпрямленного тела в голову кровь приливает в том числе, лысые обезьяны такое пиздец не любят, внутричерепное давление подскакивает, мозга туманеют. В принципе по-моему манярасчету с определением угловой скорости на глаз, когда он именно головой в направлении радиус-вектора крутился, там все же не больше 2же получается, но головушка у человеков нежная, и такое тяжело переносит, так что хиккан с сосача наверняка бы отключился и упал бы мешком нахуй. Честно сказать даже удивлен, что до такой закрутки довели, неужели не предусмотрели контрмер? Стабилизатор там какой-нибудь подвесной, на металлической цепи, с оконечником аэродинамической формы, 100 кг массойебать я у мамы инженер, ток блять ебучий стратостат не утащит столько балласта? А дистанционного открывателя парашютов ему там не организовали, лол? Вообще так-то выходит действительно рисковый прыжок и многое могло пойти не так. А я думал что весь хайп именно что из-за высоты. Не рассматривал с точки зрения риска фатальной закрутки.
Аноним 06/03/17 Пнд 13:04:03  329103
>>329099
Карочи я просто не эрудированный быдла. Почитал вот статейку
https://ru.wikipedia.org/wiki/Проект_«Эксельсиор»
И все вопросы отпали
Аноним 06/03/17 Пнд 13:11:25  329106
>>329103
Так, где там успешный пояснятель, сорвавший аншлаг в этом ИТТ треде?.. Ага, вот он>>328573
У тебя вопрос за гранью научного подхода. Современная наука вообще пока не'define'ет "небытие". Поэтому вопрос "можно" скорее к накуренным философам-теоретикам, нежели к спейсачерам.
Аноним 06/03/17 Пнд 13:13:05  329107
Промахнулся, это>>329106
тебе
>>329102
Аноним 06/03/17 Пнд 13:21:03  329109
>>329101
Ну если шоб прям по кварки, то вопрос скорее не "сколько", ибо ответ оче дохуя, по крайней мере плотность потока высокая нужна, а насколько "гамма", хотя и тут ответ в принципе очень дохуя.
Насчет полей не знаю, что там с тушкой в магнитном случится, гемоглобин из крови распидорасит или как, но в постоянном поле гравитации нужен не столько охуеть параметр самой гравитации, сколько высокий параметр градиента этой самой гравитации, который черные дыры или даже нейтронные звезды вполне обеспечат. Правда наблюдение за эпичной дезинтеграцией в таком случае может быть затруднено ввиду специфических выкрутасов метрики пространства-времени.
Аноним 06/03/17 Пнд 13:31:34  329110
>>329102
Отдельных кварков не бывает.
Аноним 06/03/17 Пнд 13:49:23  329112
>>329099
>Честно сказать даже удивлен, что до такой закрутки довели, неужели не предусмотрели контрмер?
У Баумгартнера был спортивный прыжок, суть его была в том, чтобы поставить рекорд в максимально обычной снаряге. У Юстаса, который чуть спустя прыгнул выше, был уже специальный костюм со всеми ухищрениями. Но он не ради рекорда делался, а ради инженерного челленджа и острых ощущений, скрыто и практически секретно. Контора, которую запилил Юстас для разработки технологии прыжка, потом для таких же любителей собиралась продавать прыжки по 75 килобаксов если я правильно помню, но то ли дораха, то ли подготовка нужна была ебическая, то ли мало кому интересно было, в общем они сдулись быстро.
Аноним 06/03/17 Пнд 14:00:27  329114
Посмотрел марсианина и охуел со сцены с перчаткой. Где ваще можно посмотреть ляпы этого фильма?
Аноним 06/03/17 Пнд 14:25:12  329120
>>329118
Космическая одиссея 2001))
Аноним 06/03/17 Пнд 16:31:11  329122
>>329031

Но ведь все крутится вокруг Ядра - то есть и огарок тоже с нами путешествовал же!!!

Дальше, почему мы не встречаем огарки других звезд, которые дали материал для других звезд ну и т.п.?

Все отличное Н - образовано звезде.
Все тяжелее от НЕ - образовано в большой звезде.
Все тяжелее от ФЕ - образовано в неебатца какой тяжелой звезде.

У нас в системе дохуя металла и тяжелых элементов (ну иначе были бы только всякие Сатурны и Европы онли).

Следовательно, материал нам дали сверхтяжелые звезды. Сколько светолет тяжелые элементы будут лететь? 5-10? Не более. Дальше их свяжет облако газа или гравитационный центр.
Аноним 06/03/17 Пнд 16:56:27  329125
>>329122
Немалая часть тяжелых элементов образована в результате взрыва нейтронных звезд и белых карликов в двойных системах.
Аноним 06/03/17 Пнд 17:22:54  329126
>>329096
надо будет перекатить биологического бессмертия тред в пейсач, такая-то благая почва
Аноним 06/03/17 Пнд 17:40:21  329129
02222f124f4834a[...].jpg (53Кб, 950x548)
Реально ли создать магнитное поле для Марса повесив мощный магнитик в точке Лагранжа? Закрыть весь Марс всего лишь одной космической станцией и защитить атмосферу от утекания?

это сейчас в пораше всерьез обсуждают
Аноним 06/03/17 Пнд 18:00:05  329130
>>328065 (OP)
Когда там позитронов на фотонный двигатель наскребут?
Аноним 06/03/17 Пнд 18:29:04  329131
>>329129
Ну, солнечный ветер таким способом отклонить технически возможно (т.к. он представляет собой плазму, т.е. поток заряженных частиц), но практически недостижимо (т.к. требуются очень сильное магнитное поле, даже с YBCO магнитом в разумных рамках энергобаланса и массогабаритов столько не получить. Но самое главное нахуя.
Аноним 06/03/17 Пнд 18:47:15  329132
5k5No.png (25Кб, 500x210)
>>329129
Потребуется два кольца Гельмгольца диаметром 12000 км, в точке L1. Обмотка из медного кабеля, сечением 1 кв метр, и 10 мегаватт энергии, что бы закрыть Марс целиком. Это то количество меди, которое производят на Земле за 50 лет. Занимайтесь в пораше выливанием говна друг на друга, у вас там это лучше получается.
Аноним 06/03/17 Пнд 18:48:37  329133
>>329130
Наскрести - не проблема. Проблема хранить, транспортировать и безопасно извлекать энергию.
Аноним 06/03/17 Пнд 19:03:54  329134
>>329132
Это с проводником. Сверпроводящий магнит будет во много раз меньше. криогеника этой ебалы - дело отдельное
Аноним 06/03/17 Пнд 19:14:38  329135
>>329134
Сверхпроводящий магнит в проводнике не нуждается, места не занимает, он из магии сделан.
>будет во много раз меньше
Во сколько раз меньше? Давай конкретное инженерное решение.
Аноним 06/03/17 Пнд 19:16:10  329136
>>329129
Нереально. Сдует солнечным ветром. Положение в точке Лагранжа и так нестабильно, а у ж такой здоровый магнитный парус, который способен прикрыть собой планету там точно долго не усидит. Если уж и строить генератор искусственного магнитного поля, то на самом Марсе. Но это будет абсолютно циклопическое сооружение.
Только вот он нафиг не нужен. Лучше лишние ресурсы направить на плавление полярных шапок и прочие прожекты увеличения плотности атмосферы. Толка будет больше.
Аноним 06/03/17 Пнд 19:24:34  329137
>>329135
>Сверхпроводящий магнит в проводнике не нуждается, места не занимает, он из магии сделан.
Зачем место, когда нужно поле? Сильнее поле/больше индукция - больше напряженность на больших удалениях. И зачем именно кольца Гельмгольца? Простой линейный проводник в L1, чем больше поле - тем его меньше надо.
Аноним 06/03/17 Пнд 19:25:06  329138
>>329134
А объём добычи какого-нибудь иттрия для этого сверхпроводника во сколько будет меньше?
Аноним 06/03/17 Пнд 19:42:27  329141
>>329137
Затем, что нужно поле определенного размера и напряженности.
>Сильнее поле/больше индукция - больше напряженность на больших удалениях.
>Простой линейный проводник в L1
>линейный проводник в L1
Длинна, материал.
>чем больше поле - тем его меньше надо
Объяснись.
Аноним 06/03/17 Пнд 20:18:09  329143
>>329102
закон сохранения материи/энергии не позволяет
Аноним 06/03/17 Пнд 21:26:34  329157
>>329118
1. То, что ты описываешь, называется "документальное кино".
2. Если снимать без ляпов и допущений, то кина не получится, как говорят ребята из РедЛеттерМидиа
Аноним 06/03/17 Пнд 22:03:11  329165
Зачем ракеты красят чёрно-белыми полосами/элементами/блоками либо сплошным оранжевым?
Аноним 06/03/17 Пнд 22:13:12  329168
>>329165
>сплошным оранжевым?
Потому что пенополиуретан, в качестве теплоизоляции.
Аноним 06/03/17 Пнд 22:17:50  329169
>>329165
>чёрно-белыми полосами/элементами/блоками
>>321013
Аноним 07/03/17 Втр 01:11:25  329173
Беременность в космосе. Если исключить радиацию под корень, то какие проблемы у развития плода остаются в невесомости?
Аноним 07/03/17 Втр 09:08:46  329192
>>329113
Обосрался зайдя в тред тупых вопросов про космос, а не про философию.
Аноним 07/03/17 Втр 09:25:22  329194
>>329193
Просто твой вопрос в по сути "Что первично материя иди сознание?"
Так что гугли метериализм и идеализм, я думаю тебе это будет интересно.
А наука пока говорит "синопсы в мозгу заряды туда сюда возбудимся связь встанет".

Аноним 07/03/17 Втр 13:33:52  329201
Что остается от карлика после взрыва типа 1а? Его распидорашивает полностью или сето остается? И что случается со звездой-компаньоном?
Аноним 07/03/17 Втр 14:15:51  329207
>>329201
> карлика
> 1а
ёбу дал?
Аноним 07/03/17 Втр 14:18:34  329208
>>329173
остаётся невесомость
Аноним 07/03/17 Втр 14:24:53  329209
Белого карлика Карл! Белого!
Аноним 07/03/17 Втр 14:47:51  329212
>>329209
>>329210
> белого
станет НЗ
Аноним 07/03/17 Втр 21:05:22  329258
>>329212
Может и сверхновой стать если много вещества перетянет.
>>329209
Белый карлик хотя бы раньше был нормально звездой, а вот коричневый карлик никогда уже нормальной звездой не станет. Астрофизики ебаные расисты.
Аноним 07/03/17 Втр 22:03:33  329263
>>329261
И в каком месте тут нормальная звезда? За всю историю наверное один белый карлик смог хоть как-то сравняться с нормальными звёздами. Я говорю, конечно же, про Петю Динкледжа 40 Эридана B которую открыли ажно почти 250 лет назад.
Только сейчас, с развитием мультимедиа астрономии у нас появилась возможность видеть настолько ничтожные объекты и выделять их на фоне таких гигантов, как ДЖОН СИНА Сириус и тому подобных.
Аноним 08/03/17 Срд 02:50:25  329297
Терешкова - коп[...].webm (1641Кб, 480x360, 00:00:37)
Что за странная история с нештатной ситуацией у Терешковой?
С орбиты она бодро рапортовала и пела песни и лишь много лет спустя мир узнал, что та звездная одиссея могла стоить ей жизни - в программе корабля была неточность и вместо того чтобы спускаться, с каждым витком он все дальше удалялся от Земли. Ошибку удалось исправить. http://www.1tv.ru/news/2012-03-06/96118-valentina_tereshkova_otmechaet_yubiley
На работу чего могла повлиять ошибка в программе? Ведь на космических кораблях Восток не было корректирующей двигательной установки. Были газовые ракетные двигатели, которые вращали корабль вокруг центра масс. И была тормозная двигательная установка одноразового включения. Однако, она запускалась исключительно на витке спуска. Соответственно, её возможные ошибки в работе не могли проявиться в течение допосадочных витков, во время которых корабль, якобы, удалялся от Земли. Каким же тогда агрегатом могла управлять ошибочная программа? И как понять в программе корабля была неточность и вместо того чтобы спускаться, с каждым витком он все дальше удалялся от Земли. Будто программа управления предусматривала постепенный спуск корабля в течение всего полета, а вышло наоборот. Но это не так. Программа запускала тормозную установку только на финальном витке, до него корабль не управлялся.

Непонятную ситуацию исправили непонятно как.
Версия 1
Проблема оказалась в том, что сориентировать правильно корабль космонавту не удавалось. Терешкова пыталась направить "Восток-6" к Земле, а он, получив команду, разворачивался в противоположную сторону. Была опасность, что корабль вообще не сойдет с орбиты.Поначалу инженеры все пытались свалить на "особенности женского" организма - мол, плохо Терешкова готовилась, растерялась в невесомости. Но потом выяснилось: не космонавт виновата. При сборке системы управления в ручном режиме полярность контактов была не соблюдена, поэтому корабль и уходил совсем не в ту сторону, в которую должен был. Удача, что автоматическая система управления была собрана правильно. Когда пришло время посадки, техника сработала и правильно сориентировала "Восток-6" http://www.rostov.kp.ru/daily/26088/2990084/
Терешкове не удалось сориентировать корабль, получилось у автоматики

Версия 2 вебмрилейтид. Терешкова в интервью Первому каналу Это случилось когда выведен был корабль, и я обратила внимание, что я не смогу вернуться на Землю, что корабль был сориентирован не на сталкивание корабля с орбиты, а наоборот, на подъем орбиты. Я доложила на Землю, я получила какие нужно выставить координаты, все сделала..
По версии Терешковой, корабль ориентирован самой Терешковой. Но как: верную ориентацию получила, введя(вопрос еще - куда введя?) какие-то координаты. Хотя для ручной ориентации Востоков космонавт двигал ручку управления газовыми двигателями ориентации. О каких координатах для ориентации говорит Терешкова?
Еще один момент Это случилось когда выведен был корабль, и я обратила внимание, что я не смогу вернуться на Землю, что корабль был сориентирован не на сталкивание корабля с орбиты Корабль должен ориентироваться непосредственно перед включением тормозной установки. Какая разница как он ориентирован в начале полета?
Аноним 08/03/17 Срд 03:08:53  329298
koty116511108or[...].jpg (45Кб, 500x276)
Пилите перекат уже.

Вы ставите спейсач под угрозу, ньюфаги не видя тред тупых вопросов побегут создавать свои тупые треды посвященные одному вопросу.
Аноним 08/03/17 Срд 03:37:57  329299
BlueprintVostok.jpg (440Кб, 1771x1704)
>>329297
Какие-то очень сомнительные у тебя источники, прозреваю массированное изнасилование журналистов учёными. Во всех более-менее надёжных источниках одна и та же история, вот покушай Каманина например, тот в пиздеже не замечен:
>С 10 часов готовим все данные для посадки обоих кораблей. "Чайку" будем сажать на 49-м витке (запасные витки для ручного спуска - 51-й и 54-й)
>Она дважды пыталась сориентировать корабль и честно призналась, что ориентация по тангажу у неё не получается. Это обстоятельство всех нас очень беспокоит: если придётся садиться вручную, а она не сможет сориентировать корабль, то он не сойдет с орбиты. На наши сомнения она ответила: "Не беспокойтесь, я всё сделаю утром". Связь она ведет отлично, соображает хорошо и пока не допустила ни единой ошибки. За ночь она отдохнет и автоматическую посадку должна перенести хорошо. Поручили Гагарину, Титову, Николаеву и Раушенбаху на 45-м витке потренировать её в ориентировании корабля по посадочному варианту. Ребята подготовили план переговоров с ней, согласовали со специалистами все рекомендации и попытаются ей помочь.
>Утром "Чайка" выполнила ориентацию корабля "по-посадочному" и 15 минут держала корабль в таком положении. Терешкова была очень довольна и радостно доложила о результатах пилотирования кораблем. Сергей Павлович и другие члены комиссии тоже были удовлетворены результатами её пилотирования и поверили, что в случае неприятностей с автоматической посадкой Терешкова сумеет вручную посадить корабль.
>В 9 часов 39 минут 40 секунд была выдана команда на включение автоматического цикла посадки. Через несколько секунд мы узнали, что команда прошла...Через несколько минут после расчетного времени раскрытия главного парашюта пеленгаторы засекли корабль и выдали первые координаты его приземления: "Восток-6" спускался точно по орбите 49-го витка, но со значительным перелетом...
То есть был обсёр с ручной ориентацией, нужной на случай ЧП, а корабль в любом случае сел на автоматике по плану. Откуда взяли перепрограммирование корабля и удаление от Земли с каждым витком, я вообще не понимаю. Гугл отсылает только на всякие сайты НТВ, где сильно попахивает журнашлюхами, которые нихуя не поняли и накатали отсебятины. Ну а Терешкова может ёбнулась на старости лет, хуй знает.
Аноним 08/03/17 Срд 11:14:01  329311
iss036e030730.jpg (1615Кб, 4256x2832)
Подскажите ресурс, на котором угорают по перспективным идеям в космонавтике? Электромагнитные паруса и ЭЯУ, вот это всё. Раньше таковым был раздел aftershock'а, теперь сайта нет вроде.
Аноним 08/03/17 Срд 16:45:59  329323
nikon-action-ex[...].jpg (62Кб, 744x414)
Здравствуй, космач!

Есть два бинокль Nikon ActionEX 12x50 заебись вещь, палю соседей по окнам. Живу на окраине города. С рук Луну уже смотрел, Венеру смотрел показалось даже, что вижу серп, лол. Один раз, кажется, видел какую-то слабую туманность, охуел, задумался о вечном. Скоро придет адаптер для треножника с aliexpress, чтобы все не моталось перед глазами.

Какие я смогу увидеть интересные туманности/двойные звёзды/планеты в такой бинокль?

Про Stellarium знаю, и по нему же следующий вопрос.

Можно ли в Stellarium как-то ограничить/выделить объекты в заданных мной пределах яркости? А то при зуме он мне показывает все вплоть до глубокого загло космоса.
Аноним 08/03/17 Срд 20:04:03  329336
>>329323
>туманности
Кроме Андромеды - никакие.
>двойные звёзды
Извини, но очень лень вычленять из всего невьебенного списка звезд те, на которые хватит разрешающей способности твоего бинокля.
>Можно ли в Stellarium
Можно указать в настройках придельные звездные величины, для твоего бинокля она будет около 10. Еще вариант нажать справа вверху на гаечный ключ и в поле окуляры добавить свой, поставить галку "бинокль", и в нужные поля вбить параметры своего бинокля. Далее выбираешь объект и тыкаешь справа вверху окуляр, далее прощелкиваешь мышкой до своего бинокля и получаешь, то что в теории должно быть видно.
Аноним 08/03/17 Срд 20:07:08  329337
>>329298
Делай сам, или не пизди.
ПЕРЕКОТ Аноним 08/03/17 Срд 20:24:44  329343

ПЕРЕКОТ

https://2ch.hk/spc/res/329342.html
>>329342 (OP)
Аноним 09/03/17 Чтв 16:21:01  329451
>>329336
Cпасибо!

[Назад][Обновить тред][Вверх][Каталог] [Реквест разбана] [Подписаться на тред] [ ] 525 | 70 | 101
Назад Вверх Каталог Обновить

Топ тредов
Избранное